Sie sind auf Seite 1von 41

 

For all Agricultural, Medical, Pharmacy and Engineering Entrance Examinations held across India.

NEET-UG / AIPMT & JEE (Main) 

Physics
 

 
Salient Features
 
• Exhaustive coverage of MCQs subtopic wise.
  • ‘4024’ MCQs including questions from various competitive exams.
  • Includes solved MCQs from MHT CET 2016, NEET P-I and P-II 2016,
JEE (Main) 2015 & 16, AIPMT 2015 & Re-Test.
  • Various competitive exam questions updated till latest year.
  • Concise theory for every topic.
• Neat and authentic diagrams.
 
• Hints provided wherever relevant.
  • Topic test at the end of each chapter.
• Important inclusions: Knowledge bank and Googly questions
 

 
Solutions/hints to Topic Test available in downloadable PDF format at
 
www.targetpublications.org/tp10063
   

  Printed at: Repro India Ltd. Mumbai


 
© Target Publications Pvt. Ltd.
No part of this book may be reproduced or transmitted in any form or by any means, C.D. ROM/Audio Video Cassettes or electronic, mechanical
including photocopying; recording or by any information storage and retrieval system without permission in writing from the Publisher.

P.O. No. 32715

10063_11010_JUP
PREFACE
Target’s “NEET Physics Vol-II” is compiled according to the notified syllabus for NEET-UG & JEE (Main), which
in turn has been framed after reviewing various state syllabi as well as the ones prepared by CBSE, NCERT and
COBSE.
The book comprises of a comprehensive coverage of Theoretical Concepts & Multiple Choice Questions. The flow
of content & MCQ’s is planned keeping in mind the weightage given to a topic as per the NEET-UG & JEE (Main)
exam.
MCQ’s in each chapter are a mix of questions based on theory, numerical and graphical. The level of difficulty of
these questions is at par with that of various competitive examinations like CBSE, AIIMS, CPMT, JEE, AIEEE,
TS EAMCET (Med. and Engg.), BCECE, Assam CEE, AP EAMCET (Med. and Engg.) & the likes. Also to keep
students updated, questions from most recent examinations such as AIPMT/NEET, MHT CET, K CET, GUJ CET,
WB JEEM, JEE (Main), of years 2015 and 2016 are exclusively covered.
Unique points are represented in the form of Notes at the end of theory section, Formulae are
collectively placed after notes for quick revision and Shortcuts are included to save time of students while
dealing with rigorous questions.
An additional feature of Knowledge Bank is introduced to give students glimpse of various interesting concepts
related to the subtopic.
Googly Questions are specifically prepared to develop thinking skills required to answer any tricky or higher
order question in students. These will give students an edge required to score in highly competitive exams.
Topic Test has been provided at the end of each chapter to assess the level of preparation of the student on a
competitive level.
We are confident that this book will cater to needs of students of all categories and effectively assist them to achieve
their goal. We welcome readers’ comments and suggestions which will enable us to refine and enrich this book further.
All the best to all Aspirants!
Yours faithfully
Authors

No. Topic Name Page No.


1 Electrostatics 1
2 Current Electricity 108
3 Magnetic Effect of Electric Current 181
4 Magnetism 248
5 Electromagnetic Induction and Alternating Current 302
6 Electromagnetic Waves 371
7 Ray Optics 397
8 Wave Optics 476
9 Interference of Light 494
10 Diffraction and Polarisation of Light 521
11 Dual Nature of Matter and Radiation 554
12 Atoms and Nuclei 596
13 Electronic Devices 664
14 Communication Systems 737
 
Note: ** marked section is not for JEE (Main)
01 Electrostatics
Chapter 01 : Electrostatics

      
 
 
 
 
  1.1 Electric charges and their conservation 1.12 Equipotential surface
  1.2 Coulomb’s law-force between two point 1.13 Electric potential energy of a system of two
  charges point charges and of ** electric dipoles in
  electrostatic field
1.3 Superposition principle, forces between
 
multiple charges 1.14 Conductors and insulators, Free and bound
  charges inside a conductor
  1.4 Continuous distribution of charges
  1.5 Electric field, Electric field lines, Electric 1.15 Dielectrics and electric polarization
  field due to a point charge 1.16 Capacitors and Capacitance
 
  1.6 Electric dipole and electric field due to a 1.17 Capacitance of parallel plate capacitor with
dipole and without dielectric medium between the
   
plates.
  1.7 Torque on a dipole in uniform electric field  
  1.8 Electric Flux 1.18
  Combination of capacitors in series and
    parallel
  1.9 Gauss’ theorem and its applications  
1.19 Energy stored in a capacitor
  1.10 Electric potential and **potential difference  
  **1.20
  Van de Graaff generator
  1.11 Electric potential due to a point charge, a  
dipole and a system of charges
   
   
1.1 Electric charges and their conservation  Charging by induction
i. The redistribution of charges within a
 Electrostatics: body due to the presence of a nearby
The study of electricity or electric charges at charge is called electrostatic induction.
rest is known as electrostatics. ii. The charged body induces an opposite
charge on the neutral body without
 Charge:
actually touching it.
i. The property of particles like protons and
For eg: Attraction of paper bits towards a comb
electrons which produces electrical
is due to the charging of comb by
influence is called as charge.
rubbing it against dry hair. When the
ii. It is a scalar quantity.
charged comb is brought near the paper
iii. Formula: q = It
bits, it induces an opposite charge at the
where q is charge, I is current, t is time.
end of paper nearer to the comb resulting
iv. Unit: coulomb in SI system and stat
in attraction.
coulomb in CGS system
1 C = 3  109 stat coulomb  Quantisation of charge:
stat coulomb is also called electrostatic i. Charge (q) on a body is always equal to
unit (e.s.u.) of charge. the integral multiple of electronic charge
v. Dimensions: [M0 L0 T1 A1] (e).
vi. There are two types of charges: positive  q =  ne
and negative. where n = 1,2,3,…
vii. Like charges repel, unlike charges attract. ii. Each electron bears a charge equal to
viii. The electric charge is additive in nature –1.6  10–19 C.
and is invariant.
ix. Accelerated charge radiates energy.  Conservation of charges:
x. Charge cannot exist without mass. i. For an isolated system, the net charge
xi. Charge does not experience any force due always remains constant.
to electric field produced by it. ii. Net charge can neither be created nor be
xii. A body can be charged by rubbing or destroyed in any isolated system.
conduction or induction. iii. Transfer of charges is possible.
1
Physics Vol‐II (Med. and Engg.)

1.2 Coulomb’s law  force between two 1.3 Superposition principle, forces
point charges between multiple charges

 Coulomb’s law:  Superposition principle:


i. Force of attraction or repulsion between i. Total force acting on a given charge due
two charges is directly proportional to the to number of charges is the vector sum of
product of the charges and inversely the individual forces acting on that charge
proportional to the square of the distance
due to all the charges.
between them.
q1q 2 ii. Consider number of charges q1, q2, q3….
F are applying force on a charge q
r2
qq Net force on q will be
 F = C 12 2     
r Fnet  F1  F2  ....  F n 1  Fn
where, q1 and q2 are charges separated by
distance r. q
1 r1
ii. In air, C = in SI system and C = 1 in rn
4 0
q1 r2 rn–1
CGS system. r3 qn
1 q2 qn–1
In any medium, C  in SI system q3
4k0
1 iii. The magnitude of the resultant of two
and C = in CGS system.
k electric forces is given by,
iii. The force between two charges in any
medium, Fnet
1 q1q 2 F2
F=
4k 0 r 2  
iv. In vector notation, force on q2 due to q1 is F1
given as,

1 q1q 2  Fnet = F12 + F22 + 2F1F2cosθ
F21 = r 12
4k 0 r123
F2sinθ
 and tanα =
where, r 12 is the position vector from q1 F1 + F2cosθ
to q2. iv. The force between any two charges is not
Force on q1 due to q2 is given as, affected by the presence or absence of

1 q1q 2 
F12 = r 21 other charges.
4k 0 r213

 Forces between multiple charges:
where, r 21 is the position vector from
i. Principle of superposition is used to
q2 to q1.
calculate electric force on a charge due to
v. Coulomb’s force between charges is
central force and acts along the line other charges in the vicinity.
joining the charges. ii. For N point charges q1, q2, …. , qN located
vi. Coulomb’s force between the two charges   
at positions r 1, r 2 ,.... r N with respect to
is independent of presence of other

charges in the surrounding. origin respectively, total force F1
 Dielectric constant: experienced by charge q1 due to all other
Dielectric constant is defined as the ratio of charges is given by,
permittivity of any medium () to the permittivity     
of free space (0). F1  F12  F13  F14  ........  F1N
 iii. Using vector form of Coulomb’s law,
i.e., k =
0  
  = k0       

 1 r r r r r 1 r N 
F1   q1q 2 1 2 3  q1q 3 1 3 3  ........  q l q N
where, 0 = 8.85  1012 C2/Nm2 and  40       3

1  r1  r2 r1  r2 r 1 r N 
= 9  109 Nm2/C2 or farad metre–1 (Fm–1)  
4 0

2 2
Chapter 01 : Electrostatics

1.4 Continuous distribution of charges iii. Force on charge q0 due to volume charge
distribution is,
 Continuous distribution of charges:  q0 dv
i. A system of closely spaced electric
F
40  r' 2
rˆ'

charges form a continuous charge


distribution. 1.5 Electric field, Electric field lines,
ii. On macroscopic level, quantisation of Electric field due to a point charge
charges is ignored. For a charged body
with reasonable size, its charge  Electric field:
distribution is treated as continuous. i. The space around charge in which its
iii. The continuous distribution can be electric force can be experienced is called
categorized as linear, surface and volume electric field.
charge distribution. ii. The electric field is a vector quantity
introduced as an intermediary between
 Linear charge density: charges.
i. When charge is distributed along a line, Charge  field  charge
charge distribution is called linear. iii. The electric field is characteristic of
q charge or system of charges and
ii. Linear charge density,  
L independent of the test charge placed at a
where, L is length of rod. point.
iii. Unit: coulomb metre–1 (Cm–1) iv. The electric field is quantified by electric
iv. Dimensions: [M0 L–1 T1 A1] field intensity.
 Surface charge density:
Electric field intensity  E  :


i. When charge is distributed over a surface,  
charge distribution is called surface i. Electric field intensity at any point in an
charge distribution. electric field is defined as the force acting
ii. Surface charge density, per unit test charge at that point in an
q electric field.

A ii. Electric field intensity is a vector quantity
where, A is surface area. and represents strength of electric field.
iii. Unit: coulomb metre–2 (Cm–2) 
 F
iv. Dimensions: [M0 L–2 T1 A1] iii. Formula: E  where, q0 is test charge
q0
 Volume charge density: iv. Unit: newton coulomb–1 (NC–1) or
i. When charge is distributed over the volt metre–1 (Vm–1) in SI system.
volume of an object, it is called volume v. Dimensions: [M1 L1 T–3 A1]
charge distribution. vi. It is also called as electric field strength or
ii. Volume charge density, electric field.
q vii. In the presence of dielectric, electric field

V 1
where, V is volume. decreases and becomes times its value
k
iii. Unit: coulomb metre–3 (Cm–3) in free space.
iv. Dimensions: [M0 L–3 T1 A1]
 Electric field due to a point charge:
 Force due to various charge distribution: i. Consider an isolated point charge q at the
i. Force on charge q0 due to line charge origin. Force experienced by test charge q0
distribution, 
 q0 dl
at distance r is,
40  r' 2
F rˆ'  1 qq 0
F rˆ
4 0 r 2
where, r is distance between charge
element and point under consideration. ii. The electric intensity at the point is given
r̂' is unit vector directed from charge as,
element to the point. 

ii. Force on charge q0 due to surface charge


 F
E
distribution, q0
 q0 ds  1 q
 E
F
40  r' 2
rˆ'
4 0 r 2

3
Physics Vol‐II (Med. and Engg.)

iii.

The magnitude of E follows inverse  Electric field intensity due to an electric
square law. dipole at a point on its axial line (End on
iv. The electric field due to point charge is position):
spherically symmetric. i. A line passing through the positive and
negative charges of the electric dipole is
 Electric field lines: called the axial line of the electric dipole.
i. The imaginary path along which a free ii. Consider an electric dipole consisting of
positive charge moves when placed in an two point charges ‘– q’ and ‘+ q’
electric field is called as electric lines of separated by a distance 2l as shown in
force. figure.
ii. They start from a positive charge and end
on a negative charge. Axial Line
Y
iii. The tangent to the line of force at any   

point gives the direction of the electric –q +q E1 E E2



field intensity E at that point. A O B C P D X
iv. Two lines of force do not intersect each l l
other. r
v. The lines of force are normal to the 
E at a point on the axial line
surface of a charged conductor at any
point. The medium between the electric dipole
vi. Lines of force do not pass through a and the observation point has dielectric
conductor. Hence the electric field inside a constant k.
conductor is always zero. Lines of force 

can pass through an insulator. iii. If E1 is the electric intensity at P due to


vii. Electric lines of force are crowded charge ‘– q’, then
together where the field is strong and  1 q
E1  (ˆi)
widely separated from each other where 4 0 k (r  l ) 2
the field is weak. 

viii. The lines of force are under tension and E1 is represented both in magnitude and
tend to shrink. This explains why two direction by PC .
unlike charges attract each other. 

ix. The lines of force exert a lateral pressure iv. If E 2 is the electric intensity at P due to
on one another. This explains why like charge ‘+q’, then
charges repel each other.  1 q ˆ
E2  i
4 0 k (r  l ) 2
1.6 Electric dipole and electric field due 
to a dipole E 2 is represented both in magnitude and
 Electric dipole: direction by PD .
i. System of equal and opposite charges v. Resultant intensity,
separated by a small fixed distance is E = E2 – E1
called dipole. q  1 1 
ii. Electric dipole moment p = q(2l) =    î
40 k  (r  l ) 2 (r  l ) 2 
where, 2l = dipole length
q = magnitude of either of the charge. 2(q  2l ) r 
= i
 40k (r 2  l 2 ) 2
iii. In vector form, p  q  2l pˆ
 1 2pr 

E i
The direction of p is from negative 4 0 k (r  l 2 ) 2
2

charge to positive charge. vi. If dipole is of very small length, i.e.,


iv. Unit:  1 2p 
coulomb metre (Cm) or debye in SI unit. l < < r, E  i
40 k r 3
stat C cm in CGS unit.
1 2p
v. Dimensions: [M0 L1 T1 A1] For vacuum E =
vi. It is a vector. 40 r 3
vii. The electric field produced by a dipole is vii. The direction of electric field intensity due
known as dipole field. to electric dipole at a point on its axial line
viii. The dipole field has a cylindrical is always along the direction of the dipole
symmetry. moment.
4 4
Chapter 01 : Electrostatics
 Electric field intensity due to an electric  1 p

dipole at a point on the equatorial line (Broad viii. In vector form, E 


4 0 k r 3
side – on position):  
i. Equatorial line of an electric dipole is a Negative sign indicates E and p are in
line perpendicular to the axial line and opposite direction.
passing through a point mid-way between 1 p
For vacuum E =
the charges of the dipole. 4 0 r 3
ii. Consider an electric dipole consisting of ix. Using parallelogram law of vectors,
two point charges ‘– q’ and ‘+ q’ E = 2 E1 cos 
separated by distance 2l as shown in i.e., Eaxial = 2 Eequator
figure.  Electric field intensity at a general point due
D to short electric dipole:

 E2 i. Consider a short electric dipole with


E P

R dipole moment p placed in vacuum. Let


O be the mid-point of dipole and line OP
C E1

makes an angle  with p .
r2  l 2 r r2  l 2 
E2
B E  A


 E2
 E1
–q +q P
A O B (r, )
l l r

E at a point on equatorial p cos 

iii. The medium between the electric dipole –q 
+q
O p
and the observation point has dielectric
constant k. p sin 
iv. If E1 is the electric field intensity at P due
to charge ‘– q’, then ii. For any general point P, axial line
1 q component of electric field intensity is,
E1 =
4 0 k  r  l 2 
2
E1 
1 2p cos 
along PA.
4 0 r3

E1 is represented both in magnitude and Equatorial line component of electric field
direction by PC . 1 psin 
intensity is, E 2  along PB.
v. If E2 is the electric field intensity at P due 40 r 3
to charge ‘+ q’, then iii. Resultant electric intensity is,
1 q  1 p
2
E2 = (4 cos2 + sin2)
4  0 k  r  l 2 
2 E 2  E12  E 22   3
 4 0 r 
( BP2 = OP2 + OB2) 1 p
 E 3cos 2   1
E 2 is represented both in magnitude and 4 0 r 3

direction by PD . iv. From the above figure,


E2 tan 
vi. Resultant intensity E at P is,

tan  = =
E1 2
E = E1 cos  + E2 cos 
 = tan–1  tan  
1
1 q  2l  l 
=  cos  2 
2 3/ 2  
4 0 k (r  l )
2
 r l 
2 2
v. Special cases:
1 p a. Case I: When P lies on the axial
=
4 0 k (r  l 2 ) 3/2
2
line of the dipole,  = 0.
vii. If dipole is of very small length, i.e., 1 p
E= 3cos2   1
l < < r, 4 0 r 3
1 p 1 2p
E= E=
40 k r 3 40 r 3

5
Physics Vol‐II (Med. and Engg.)
tan 0 vi. Unit:
tan  = =0
2 Nm in SI system
  = 0 dyne – cm in CGS system
b. Case II: When P lies on the vii. Dimensions: [M1 L2 T–2 A0]
equatorial line of the dipole,
1.8 Electric flux
 = 90.
1 p  Electric Flux:
E= 3cos 2 90  1
4 0 r 3 i. The number of electric lines of force
1 p passing through a given area is called as
E=
4 0 r 3 electric flux.
tan 90 ii. It is a scalar quantity.
tan  = = iii. The electric flux , through a surface
2
q
  = 90 enclosing ‘q’ is,  =
0
1.7 Torque on a dipole in uniform electric field iv. Unit: Nm2/C or V-m in SI system.
dyne-cm2/statcoulomb in CGS system.
i. An electric dipole consisting of two charges
v. Dimensions: [M1 L3 T3 A1]
+ q and – q separated by distance 2l is held in
 vi. Electric flux is the dot product of electric
a uniform external electric field E at an angle intensity vector and surface area vector,
 as shown.   ds

 =  E.ds
B F
 =  E ds cos
+q 

2l 
p 
E vii. For a closed body, outward flux is taken to
be positive and inward flux is negative.
 A  viii. Electric flux per unit area is called the flux
–F density.
–q C
ix. Electric flux is maximum when electric field
is normal to the area ds
Dipole in uniform electric field i.e., d = Eds.
ii. Two equal and opposite forces act on dipole, x. Electric flux will be minimum when electric
  field is parallel to the area
| F | |  F |  qE i.e., d = zero.
Hence net force is zero.
 Tube of force:
iii. Net torque  acts on dipole about an axis
i. The number of lines of
passing through the mid point of dipole.
force grouped together to
 = F  2l sin
form tube like structure is
= qE  2l sin called as tube of force.
  = pE sin
  
iv. In vector form,   p  E
Tube of force
v. Special cases:
a. Case I: ii. A tube of force originating from unit
When  = 0, then  = pE sin 0 = 0 positive charge is called unit tube of force
The electric dipole is in stable or Faraday’s tube of force.
equilibrium. iii. Tube of force has the same properties as
b. Case II: line of force.
When  = 90, then  = pE sin 90 iv. The number of tubes of force originating
= pE (maximum value) 1 1
from unit positive charge is = .
c. Case III:  0k
When  = 180, then  = pE sin 180 = 0 v. The number of tubes of force originating
The electric dipole is in an unstable q
from charge q is .
equilibrium. 0k

6 6
Chapter 01 : Electrostatics
 Tube of induction: E
i. Tube of force irrespective of permittivity E2 E1
of medium is called tube of induction.
ii. Only one tube of induction originates
from unit positive charge whatever may P
be the surrounding medium. r
P1 O P2
 Normal Electric Induction (NEI):
i. The number of tubes of induction passing + + + + + + + + + + + + ++ + + + +
normally through unit area is called as
normal electric induction. ii. The electric field of every point in plane
ii. Formula: normal to wire is radial and its magnitude
NEI = E = k0E depends only on radial distance r.
iii. Unit: iii. Consider a circular closed cylinder of
radius r and length l with infinitely long
C/m2 in SI system and
line of charge as its axis as shown in
stat C/cm2 in CGS system figure.

 Total Normal Electric Induction (TNEI): E
i. TNEI is defined as number of tubes of n̂ n̂
induction passing normally through a n̂
given area. P
ii. The number of tubes of induction passing ds
n̂ r n̂
normally through charge ‘q’ is
+ + + + + + + + +
q
N=
k0
n̂ n̂
iii. No. of tubes of induction passing normally
through unit area is k0E, l
TNEI =  k 0 E ds cos iv. Contribution of curved surface of cylinder
 
=  k 0 E ds towards electric flux,
 
iv. Unit: coulomb in SI system statcoulomb s E  ds = E (2rl)
in CGS system
v. Dimension: [M0 L0 T1 A1] where (2rl) is area of curved surface of
cylinder.
1.9 Gauss’ theorem and its applications v. Total electric flux through cylinder,
 Gauss’ theorem: E = E (2rl)
i. Total flux through a closed surface is Charge enclosed in cylinder q = l.
1 q
equal to times the total charge vi. From Gauss’ theorem, E =
0 0
enclosed by that surface. l
  q  E (2rl) =
0
ii.  E  ds  0

iii. Gauss’ theorem holds good for any closed  E=
20 r
surface irrespective of its shape or size.
iv. The imaginary surface which encloses the vii. The electric intensity is inversely
charge or charged body is called 1
proportional to the radius r. E 
Gaussian surface. r
viii. If  > 0, direction of electric field at every
 Application of Gauss’ Theorem: point is radially outwards.
Electric field due to an infinitely long straight If  < 0, direction of electric field at every
wire: point is radially inwards.
i. Consider an infinitely long thin wire with 

uniform linear charge density . Let P1 In this case, E is due to charge on entire
and P2 be line elements placed at equal wire while charge enclosed by Gaussian
distance on either side of origin ‘O’. surface is only   l.
7
Physics Vol‐II (Med. and Engg.)
Electric field due to uniformly charged iii. For field outside shell (r > R):
infinite plane sheet: According to Gauss’ theorem,
i. Consider thin infinite charged plane sheet   q
with surface charge density . s E ds   0
ii. Let P be any point at perpendicular q
distance r from sheet. E · (4r2) =
0
iii. Imagine a cylinder of length 2r and cross
sectional area ds around P as shown. q
E= 
4 r 2 0 E
n̂ + n̂ r
+ +

+ + P
+ +
E E r ds
+
+ ++ S1
n̂ Q
+ + P R
+ +q
n̂ + O
r +
+ ++ r n̂
+

iv. Total electric flux over entire surface of


cylinder, Thus, for a point outside the spherical
shell, field due to uniformly charged shell
E = 2 E ds
acts as if entire charge of shell is
v. Total charge enclosed by cylinder,
concentrated at the centre of the shell.
q =  ds. iv. At a point on the surface of the shell
vi. From Gauss’ theorem, (r = R):
q
E = 2 E ds = E=
q
0
4  0 r 2

E= For surface charge density ,
2 0 q = 4R2 · 
vii. E is independent of r 4R 2 
  E= = = constant
For  > 0, E is uniform, normal and 4R 0
2
0
outwards from sheet. v. For field inside shell (r < R):
 
For  < 0, E is uniform, normal and E

inwards from sheet.
P
viii. If infinite plane sheet has uniform r
ds
thickness, O R
  
E = E1 + E2 = + = S2
2 0 2 0 0

 As charge inside a spherical shell is zero,
the gaussian surface encloses no charge.
The Gauss theorem gives,
q
E1 E  4r2 = =0
0
E2
E=0
Thus, the field due to a uniformly charged
spherical shell is zero at all points inside
shell.
 
vi. Variation of E vs r
Electric field due to uniformly charged Y
spherical shape:
i. Consider a thin spherical shell of radius R Emax
E
with centre O. Let a charge + q be
distributed uniformly over the surface of 1
E
shell. r2
ii. Consider a point P, such that OP = r,
O r=R X
where r is radius of imaginary sphere S
with centre O. Distance from centre (r)
8 8
Chapter 01 : Electrostatics

1.10 Electric potential and potential difference i. Potential at point A when a charge (q0) is
brought from ,
 Electric potential: WA
VA =
i. Electric potential at any point in an q0
electric field is defined as work done in Charge +q situated at point O produces
bringing a unit charge from infinity to that electric field.
point against the direction of electric field ii. Due to presence of electric field, force on
intensity. 

ii. It is a scalar quantity. test charge is  q 0 E . To move charge


iii. Formula: against this force without acceleration
W external force is needed, which equals
V=  
q0 F  q0 E
1 q iii. If q0 is displaced through small distance dl
V=
4k 0 r then small work done is given by,
where ‘V’ is electric potential at a distance
dW =   q 0 E   dl = q0Edl = – q0Edr
 

‘r’ from charge ‘q’.  


iv. Unit: volt or JC–1 in SI system Negative sign indicates distance r
statvolt in CGS system
decreases in the direction of dl
(Electrostatic unit of electric potential)
iv. Electric field is given by,
1 volt = 1/300 statvolt
1 q
v. Dimensions: [M1 L2 T3 A1] E=
40 r 2
 Potential difference:  1 q qq 0
i. Work done in bringing a unit charge from  dW   q dr =
2  0
dr
 4  r  4  0r
2
one point to another point against the 0

direction of electric intensity is called as v. Work done in moving a charge from  to


potential difference. A is,
ii. If VA and VB are electrostatic potentials at A
1 q q0
A and B respectively, then potential WA =  dW 
 40 rA
difference between them is,
Potential at distance rA is,
WAB
V  VB  VA  1 q
q0 VA =
4 0 rA
where, WAB is work done in moving test
vi. Electrostatic potential at any point at
charge q0 from point A to B.
distance r from charge + q is,
 Relation between electric intensity and V=
1 q
potential difference: 40 r
i. When the field is not uniform 1 1
dV vii. For a single charge, F  2
; E  2 but
E=  r r
dx 1
where dV is small change in potential over V
r
small distance dx.
q
V viii. At r = , V = =0
ii. When field is uniform E = 
d
i.e., electrostatic potential is zero at
where V is potential difference over infinity.
distance d.
1.11 Electric potential due to a point charge,  Electric potential due to system of charges:
a dipole and a system of charges qn
q1
 Electrostatic potential due to a point charge: rn
  r1 qi
F   q 0E
r2 ri
 q2
+q q0 q0E A P
 r3
O A Q dl P
rA
r q3

9
Physics Vol‐II (Med. and Engg.)
i. Electrostatic potential at point P due to ii. Potential at point P due to – q
charge q1 at a distance r1 is, 1 q
V1 =
1 q 4 0 PA
V1 =
4 0 r1 And potential at point P due to + q,
ii. Total potential of point P due to the 1 q
V2 =
system of charges is given as sum of the 40 PB
potential at that point due to individual iii. Total potential at a point P is given as,
charge. V = V1 + V2
V = V1 + V2 + V3 + ……. + Vn 1 q 1 q
1 q1 1 q2 1 qn = +
=   ...........  4 0 PA 40 PB
4 0 r1 4 0 r2 4 0 rn
q  1 1 
 q1 q 2 qn  = 
40  PB PA 

1
=    ...........  
40 r1 r2 rn 
From the figure, PB = r – a cos 
n
1 q PA = r + a cos 
 V=  i
4 0 i 1 ri 1 2a cos 
V= q 2 2 2
iii. a. Electrostatic potential due to 4 0 r  a cos 
discrete charge distribution is given 1 p cos 
 V=
by, 40 (r 2  a 2 cos 2 )
1 n qi where,
V=   
40 i 1 | r  r | p is electric dipole moment = (2a) q
i

b. For linear charge distribution, iv. Special cases:


1  dl a. When  = 0, cos  = 1
V=
40   
Vaxial =
1 p
L | r  r' | 4 0 (r  a 2 )
2

where,  is linear charge density For  = , cos  = – 1


and r is position vector.] 1 p
Vaxial = –
c. For surface charge distribution, 4 0 (r 2  a 2 )
1 ds b. For a < < r,
V=
40   
p
S | r  r | Vaxial = 
4 0 r 2
where  is surface charge density
d. For volume charge distribution, c. When  = 90, cos 90 = 0
1 dv Vequatorial = 0
V=
40    v. Electric potential varies inversely as the
V | r  r | square of distance from the centre of
where  is volume charge density. dipole.
1
 Electric potential at any point due to an V
electric dipole: r2
P 1.12 Equipotential surface

 Equipotential surface:
i. Any surface, which has same electrostatic
potential at every point is called an
equipotential surface.
r ii. All the points are at same potential. So, no
work will be done in moving a charge
N from one point to another point on the
surface.
–q 
 +q 

A O B iii. E is electric field and a test charge q0 is



M moved with displacement dx on surface,
a a
then
   
i. Two equal and opposite charges + q and q0 E . dx = 0 or E . dx = 0
– q separated by distance 2a form an 
electric dipole. dx is along the surface.

10
10 
Chapter 01 : Electrostatics
 Properties of equipotential surface: Infinite Cylindrical
i. No work is done in moving a test charge linear charge equipotential
surface
over an equipotential surface. As distribution
displacement is along the surface, force
applied on it must be perpendicular.
ii. The electric field is always at right angle
to the equipotential surface.
iii. Region of strong field and region of weak Infinite line
field can be determined with the help of of charge q
equipotential surface. Electric Plane
Plane
From potential gradient, dipole equipotential
dV surface
E=
dr
dV
 dr = 
E q +q
1 Electric
For same value of dV, dr  . dipole
E
Thus, electric field will be stronger where Infinite sheet Plane
equipotential surfaces are close and of charge + +
+ q
electric field will be weaker where + +
equipotential surfaces are far. +
+ +
iv. Equipotential surfaces gives the direction +
of the electric field. Sheet
v. Equipotential surfaces cannot intersect Equipotential
surface
each other.
Shapes of equipotential surface due to various
charge distributions: 1.13 Electric potential energy of a system of
two point charges and of electric
Shape of dipoles in electrostatic field
Charge
Figure Equipotential
distribution  Electric potential energy:
surface
i. The work done in displacing a positive
Point charge Equipotential Spherical
charge (q0) from infinity to a distance r
surface from the charge q is called electric
potential energy of test charge. It is
denoted by U.
q ii. Formula:
1 qq 0
U=
Spherical 40 k r
equipotential Spherical
shell of surface iii. Unit: joule in SI system
charge (q) erg in CGS system
+ + +q Also expressed in electron volt (eV)
+ 1 eV = 1.6  1019 J
+
+
+ iv. Dimensions: [M1L2T2A0]
v. It is a scalar quantity.
spherical vi. The change in electric potential energy of
shell the system is equal to the negative of work
Non- Spherical done by electric forces.
equipotential
conducting
surface  Electric potential energy of a system of two
sphere of
charge (q) + ++ point charges:
+q + + i. The electric potential energy of a system
+ ++
of two point charges is defined as the
amount of work done to assemble this
sphere of system by bringing charges in from an
charge q
infinite distance.
11
Physics Vol‐II (Med. and Engg.)
ii. Let q1 and q2 be two charges brought in at 1.14 Conductors and insulators, Free and
positions P1 and P2 respectively from bound charges inside a conductor

infinity such that P1P2 = r 12 as shown.
 Conductors:
+q2 i. In conductors, electric charges are free to
(AT INFINITY)
move throughout the volume.
ii. The electric field inside a charged
conductor is zero.
iii. The surface of a charged conductor acts as
an equipotential surface.
iv. The total charge of a charged conductor
+q1  +q2 lies on the outer surface of the conductor.
P1(r1 ) 
P2(r2 ) v. Electric lines of force are perpendicular to
r12
the surface of a charged conductor at its
iii. The electric potential at P2 due to q1 is, every point.
1 q1 vi. If there is a cavity in a conductor or it is
V= hollow, the electric field inside the cavity
4 0 r12
is zero. This vanishing of electric field
iv. Work done in carrying charge q2 at P2,
inside the cavity of a conductor is known
W = potential  charge as electrostatic shielding.
q1 vii. The force dF acting on a small element of
=  q2
4 0 r12 area dS of the conductor where the charge
v. This work is stored in the system of two density is , is given by
point charges q1 and q2 in the form of 2
dF = dS.
electric potential energy. 2 0
q1q 2 viii. Mostly all metals are conductors.
U=W=
4 0 r12
 Insulators:
 Electric potential energy of an electric dipole i. In insulators, charges remain fixed at their
in uniform electric field: place. Thus insulators do not have free
i. The potential energy of a dipole in charges.
uniform electric field is defined as the ii. An insulator may behave in two ways:
work done in rotating a dipole from a a. It may not conduct electricity at all
direction perpendicular to the field to the and is called insulator. Mostly non-
given direction. metals are insulators.
ii. U = (W  W90) b. It may not conduct electricity
through it but on applying electric
 U = pE (1  cos )  pE
field, induced charges are produced
 U =  pE cos  on its faces. Such an insulator is
 
 U =  p.E called dielectric.
Examples: Water, mica.
iii. For  = 180, potential energy is
maximum.  Free and bound charges inside a conductor:
For  = 90, potential energy is zero. i. In conductors, valence shell is filled less
For  = 0, potential energy is minimum. than half.
iv. The variation of potential energy as a ii. Due to tendency of an atom to have a
function of  is shown below filled valence shell, the valence electrons
in atom of conductor leave atom and move
pE Umax freely through lattice of conductor.
iii. The positive ions remain fixed in their
positions.
iv. The average velocity of free electrons in
conductors is zero.
90 180  v. On the application of electric field across
the conductor, free electrons experience
force. They drift against direction of
 pE applied field constituting a net flow
through the conductor.
12
12 
Chapter 01 : Electrostatics

1.15 Dielectrics and electric polarisation  electric field between 


 
E the plates with air
 Dielectrics: ii.    k
E  electric field between 
Dielectrics are (non-conducting) material that  
transmit electric effect without conducting.  the plates with medium 
There are two types of dielectrics: iii. k is called as relative permittivity of the
i. Polar dielectrics material or SIC (specific inductive
ii. Non-polar dielectrics capacitance) and k > 1.

 Polar dielectrics: 1.16 Capacitors and Capacitance


i. It has permanent electric dipole moment  Capacitor:
even if the electric field is absent. i. Capacitor is a device which increases
ii. Net dipole moment of polar dielectric is storing capacity of a conductor at a
zero because polar molecules are randomly relatively low potential.
oriented in absence of electric field. OR
– + An arrangement which consists of two
+ – + – +– conductors separated by a dielectric
– –
+ – + + –+ medium such that one of the plate is
– positively charged and other is connected
iii. In presence of electric field, polar to the ground is called as capacitor.
molecules align themselves in the It is also called as condenser.
direction of the field. ii. Principle of a capacitor: The capacity of
Examples: Alcohol, Water, NH3, HCl. a charged conductor increases if another
 Non-polar dielectrics: conductor connected to earth is kept near
i. Every molecule has zero dipole moment it.
in its normal state. iii. Types of capacitor:
ii. When electric field is applied, molecules a. Parallel plate capacitor:
become induced electric dipole. + + + + + + +
Examples: N2, O2, Methane, Benzene.       
 Electric polarisation:
i. The process of inducing equal and k0A
opposite charges, in the two faces of C=
d
dielectric when external electric field is Cm = k Cair
applied is called electric polarisation. It is
expressed in Cm2. b. Spherical capacitor:
 r1r2 
C = 4k0  
+  r2  r1 
– + – + –
+
– + – + –
+ Ei + r+ +
+ – + – + – + 2 r1 +
+ – + – + – +
– +
+ – + – + + ++
E
ii. Electric field gets modified in presence of c. Cylindrical capacitor:
dielectric, r1 r2
E = E  Ei
Where E is induced field, + +
E is main field. + +
Ei is field due to dielectrics + +
l
 Dielectric constant: + +
i. Dielectric constant of dielectric medium is + +
the ratio of the strength of the applied
electric field to the strength of the reduced
2k 0l
value of the electric field on placing C=
r 
dielectric between the plates of the 2.303log  2 
capacitor.  r1 
13
Physics Vol‐II (Med. and Engg.)
iv. A capacitor acts as a small reservoir of iii. Since d is small, electric field between
energy. plate is uniform. The medium between
v. Applications of capacitors: plate is air or vacuum.
a. For tuning in radio circuit. P Q
b. For smoothing rectified current in +q
power supplies. B + + + + + + + + + + + + + h
c. For eliminating sparkling of points, ds
when they are open or close in an R S Gaussian
ignition system of automobile d
surface
engine. E E
d. For storage of large amount of
A             
charge in nuclear reactors.
q
 Capacitance:
iv. The flux through PR and QS of gaussian
i. It is defined as the amount of charge
surface will be zero as electric field and
required to raise the potential through 1 V.
area are perpendicular to each other.
OR
The flux through RS is given by,
The ability of a conductor to store charge  
is called as capacitance of conductor. =  E.ds =  Eds cos   EA
OR From Gauss’ law,
The ratio of charge on a conductor to its q q
corresponding potential is called EA = or E =
0 0A
capacitance of conductor.
ii. It is a scalar quantity. v. Potential difference between the plates is
q given by,
iii. Formula: C = V
V E=
iv.Unit: d
farad in SI system qd
V = Ed =
statfarad in CGS system 0 A
(i.e., electrostatic unit of capacity) q q A
vi. C= = = 0
1farad = 9  1011 statfarad V (qd / 0 A) d
1 F = 106 F vii. Capacitance of parallel plate capacitor
1 nF = 10–9 F without dielectric is given by,
1 pF = 1012 F 0 A
v. Dimensions: [M1 L2 T4 A2] C=
d
vi. Capacitance of a conductor is small and  Capacitance of parallel plate capacitor with
limited. dielectric slab between the plates:
vii. If a conductor is placed near a charged +q
conductor, the potential of charged A
conductor decreases and hence it can store P +++ + + + + + + + + + + + Q
more charge i.e., nearness of a conductor S R
increases the capacitance of a charged E0
conductor. S R t d
viii. In a capacitor, it is possible to deposit still q E
larger charges on first plate in presence of
induced negative charge on the second           
plate. B
P Q
1.17 Capacitance of parallel plate capacitor
with and without dielectric medium i. There are two capacitors with plate area A
between the plates and separated by a distance d.
 Capacitance of parallel plate capacitor ii. The dielectric slab of thickness t and
without dielectric medium: dielectric constant k is inserted between
i. Two plates of area A, kept parallel to each the plate.
other at a distance d apart and connected Electric field between the plates is given
to battery. by,
ii. Charge + q will appear on one plate which q
E=
will induce a charge  q on another plate. k 0 A

14
14 
Chapter 01 : Electrostatics
iii. Potential difference between the plate is, ii. Potential across capacitors are V1, V2 and
V = E0 (d  t) + Et V3.
q q iii. Total potential across the capacitors is V,
= (d  t) + t
0A  0 kA V = V1 + V2 + V3
q q q q
where, E 0  electric field when k = 1 iv. V1 = , V2 = , V3 = ,
0 A C1 C2 C3
q q
E electric field in medium with V=
0kA CS
dielectric constant k. where, CS is equivalent capacitance in
q  t series.
 V= (d  t)  k 
0 A   q q q q
= + +
q   1  CS C1 C2 C3
 V= d  t 1  k  
0A    1 1 1 1
 = + +
q CS C1 C2 C3
iv. C=
V
q 0 A 0A  Capacitors in parallel:
C= = C
  1   1
d  t 1   +q1 1 q1
q d  t 1   
  k   k
v. When dielectric slab is completely filled
C2
i.e., d = t
0 A  Ak +q2 q2
C= = 0 = kC0
kd  d  kd  d
C3
q q
Knowledge Bank +q3 q3

Dielectrics in capacitors serve three purposes:


i. To keep the conducting plates from + 
K
coming in contact, allowing for smaller V
plate separations. Thus, resulting in i. In the given figure three capacitors are
higher capacitance. connected in parallel.
ii. To increase the effective capacitance by
ii. Potential across each capacitor is V.
reducing the electric field strength.
iii. Charges on each plates of capacitors are
Therefore, we can get the same charge at
q1, q2, q3.
a lower voltage.
iv. Total charge
iii. To reduce the possibility of shorting out
q = q1 + q2 + q3
by sparking during operation at high
C1 C C C
voltage. q1 = , q2 = 2 , q3 = 3 and q = P
V V V V
where, CP is equivalent capacitor in
1.18 Combination of capacitors in series
parallel.
and parallel
 CP = C1 + C2 + C3
 Capacitors in series:
1.19 Energy stored in a capacitor
A B C D E F CS
i. When battery is connected across the two
+q q +q q +q q
plates of the capacitor, work is done by
V1 V2 V3 V the battery to charge the capacitor.
= ii. Charging of capacitor increases the
V potential across plates of the capacitor.
iii. The work done by the battery in charging
( ) 
( )
+  + K
a capacitor is stored in the capacitor in the
K
form of electric potential energy.
iv. Electric potential energy is given by,
i. In the given figure, three capacitors are 1 q2
U=
connected in series with each other. 2 C

15
Physics Vol‐II (Med. and Engg.)
Where, q is charge on plates of the iii. It consists of large hollow metallic sphere S
capacitor mounted on two insulating columns M1 and
C is capacitance M2.
U is electrical potential energy. iv. A long narrow belt made up of insulating
v. Energy stored can also be given as, material passes over the pulleys P1 and P2.
1 1 v. There are two metal combs C1 and C2, which
U= qV = CV2
2 2 are called as spray comb (C1) and collector
vi. When two charged capacitors are comb (C2).
connected together, there is a chance of vi. C1 is at positive potential of about
loss of energy, which can be given by 10,000V by using (HT) source, C2 is
equation, connected to sphere.
C1C2  V1  V2 
2
vii. D is an evacuated accelerating tube having an
U = U  U =
2  C1  C2  electrode I at its upper end, which is connected
to sphere S.
vii. U is positive so there is always loss of
energy. This loss is usually in the form of viii. To prevent leakage of charge from spray,
heat. generator is enclosed in steel chamber filled
with nitrogen or methane at high pressure.
1.20 Van de Graaff generator ix. When spray comb is maintained at high
positive potential it produces charges in its
i. Van de Graaff generator is an electrostatic vicinity. The positive charges get sprayed on
generator which can generate very high belt due to repulsive action of C1, which are
potential of the order of 5  106 V. It is used to then carried towards C2 by moving belt.
accelerate charged particles to carry out
nuclear reactions. x. Collecting comb C2 is positioned near the
upper end of the belt such that pointed ends
ii. Principle: touch the belt and other end is in contact with
It is based on two electrostatic phenomena: the inner surface of metallic sphere.
a. A sharp pointed conductor has large xi. C2 collects the positive charge and transfers to
charge density. Hence the surrounding outer surface of metallic sphere.
air becomes conducting and produces xii. As belt goes on moving, accumulation of
discharge called Corona discharge. positive charge on sphere keeps on taking
b. When a charged conductor is brought in place and raises the potential.
contact with hollow conductor, it
xiii. With increase of charge on the sphere, its
transfers the charge to hollow
leakage due to ionisation of surrounding air
conductor. The transferred charge
becomes faster. When the rate of loss of
resides on the outer surface of the
charge due to leakage becomes equal to the
hollow conductor.
rate at which the charge is transferred to the
sphere, the potential of sphere becomes
Steel maximum.
chamber
xiv. The projectiles such as protons, deuterons are
Metallic S introduced in the upper part of the evacuated
comb accelerator tube. They get accelerated along
the length of tube in downward directions and
P2 Metallic come out with high energy and hit the target
C2 sphere with large K.E. and bring about nuclear
D disintegration.
I
Insulating  Applications:
M1 M2 belt i. It is used to produce high potential of the
C1 order of few millions of volt.
HT ii. It is used to accelerate charged particles
P1 Motor such as proton, deuteron.
driven pulley
iii. It is used in nuclear physics for studies.
iv. In medicine, such beams are used to treat
Target cancer.

16
16 
Chapter 01 : Electrostatics

Notes 17. If electric field intensity is same both in


magnitude and direction throughout then
1. The electric charge exists on material bodies electric field is said to be uniform. Uniform
which occupy space. Hence electric charge is electric field is represented by equispaced
not a point. parallel lines.
2. The quantisation effect of charge can be 18. Total charge of electric dipole is zero but
observed only at microscopic level. electric field of electric dipole is not zero.
3. The charge can be created or destroyed in equal 19. The atom consisting of positive and negative
and unlike pairs only. charges is not a dipole, as centre of positive and
Examples: pair production, pair annihilation. negative charges coincide i.e., 2l = 0.
4. The electric charge always resides on the outer 20. The atom placed in electric field becomes dipole
surface of the charged conductor. because positive and negative centres are
displaced relative to each other.
5. Coulomb’s law is valid only for stationary point
charges and for particle separation ranging 21. At all points other than those on axial and
from 10–15 m and above. equatorial lines, the field will curve towards or
away from the charges.
6. We can use superposition principle for
computing (i) net force (ii) net field (iii) net flux 22. When electric dipole is placed in non-uniform
 
(iv) net potential and (v) potential energy at the electric field, p lies along E .
observation point P due to any configuration of
charges. 23. The value of electric flux is independent of the
distribution of charges and the separation
7. Two charges Q are at distance r from each between them inside the closed surface.
other. If third charge q(= – Q/4) is placed at a 24. Gaussian surface cannot pass through any
distance of (r/2) from each charge, then system discrete charge.
of charges will be in equilibrium.
25. If wire is not infinitely long, the end effects need
8. At the microscopic level, charge distribution is to be considered.
discontinuous. 26. For a finite sheet, edge effects need to be
9. Conceptually, continuous charge distribution is considered.
analogous to the continuous mass distribution in 27. Potential due to a point charge q at its own
mechanics. location is not defined.
10. For a surface charge distribution, electric field 28. Value of V is same at all points at equal
is discontinuous across surface. distances from a point charge. Hence potential
due to a single charge is said to be spherically
11. For a volume charge distribution, electric field symmetric.
is defined at any point in distribution.
29. The potential due to a dipole is axially
12. The electric field, as per theory, extends upto 
symmetric about p .
infinity.
32. Electric potential energy of a system of N point
13. A charge does not experience any force due to
charges is,
its own electric field.
1 j  N k  N 1 q jq k
14. If test charge has significant value, it will
U= 
2 j 1 k 1 40 rjk
k j
change, value of electric field to be measured.
 
33. The maximum electric field that a dielectric
15. For + q, F and E are in same direction i.e., medium can withstand, without breaking down
perpendicular to surface directing outwards. of its insulating property is called its dielectric
  strength.
For – q, F and E are in opposite direction and
 34. The capacity of conductor depends on its shape
E is perpendicular to surface directing inwards. and size, presence of other charged conductor
 placed nearly and nature of surrounding
16. At a neutral point, E = 0. medium.
17
Physics Vol‐II (Med. and Engg.)
35. As the potential of the earth is assumed to be 7. Electric field due to a dipole:
zero, capacity of conductor connected to earth For endon (axial) position:
(whatever its shape or charge on it) will be 1 2pr
infinite. i. E=
4 0 k (r  l 2 ) 2
2

36. A capacitor allows ac but blocks dc. ii. For broad side-on (equatorial) position:
1 p
37. Even if the current through capacitor is zero, a E=
finite amount of energy can be stored in a 4 0 k (r  l 2 )3/2
2

capacitor. iii. For any general point:


1 p
38. If space between the plates is completely filled E= 3cos 2   1
40 k r 3
with a conductor then, k =  and hence C = 
39. Series combination is useful, when a single 8. Angle between resultant electric field
capacitor is not able to tolerate a high potential intensity and electric field intensity at a point
drop. So it is distributed among number of due to axial component:
capacitors. 1 
 = tan–1  tan  
2 
40. Parallel combination is useful when high
capacity is required at low potential. 9.  = pE sin
41. Capacitor actually stores electric energy in the  

form of electric field between the plates. The net 10.  = E ds cos =  E.ds
charge on the capacitor is zero.
11. Gauss theorem:
42. The potential energy stored in the capacitor is   q
independent of the manner in which charge  E.ds    0
configuration of the capacitor is built up.
12. Electric field due to:
43. The potential energy of capacitor lies in the 
dielectric medium between the plates. i. infinitely charged wire, E =
20 r
44. The potential energy of capacitor is obtained at 
the cost of chemical energy stored in the battery ii. infinitely charged plane sheet, E =
0
used for charging the capacitor.
iii. Charged spherical shell,
45. Accelerated particles in Van de Graaff a. For a point outside the shell
generator are called projectile. q
E=
46. Van de Graaff generator is dangerous to handle 4 r 2 0
due to high potential difference. b. For a point on the surface of the

Formulae shell, E =
0
1. q = It c. For a point inside the shell E = 0

1 q1 q 2 13. Electric potential:


2. F = i. When the field is not uniform,
4 0 r 2
dV
    
E= 
dx
3. Fnet  F1  F 2  .......  F n 1  Fn
ii. When field is uniform,
Resultant of two electric forces, V
 E=
F net = F  F  2F1 F2 cos 
1
2 2
2 d
F2 sin  14. Potential due to a point charge:
tan  
F1  F2 cos  1 q
V= 

4 0 r
 F
4. E 15. Potential due to a system of charges:
q0
1 n qi
5. Electric field due to a point charge:
V= 
4 0 i 1 ri
1 q
E= 16. Potential due to an electric dipole:
4 0 r 2
1 pcos 
V=
6. p = q (2l) 4 0 (r 2  a 2 cos 2 )

18
18 
Chapter 01 : Electrostatics
17. Potential energy: 5. Electric field due to continuous distribution of
i. For a point charge,  
charge is, E   dE
1 qq 0
U= 
40 k r where dE is electric field due to a point charge.
ii. For system of two charges, 6. Direction of the torque is given by right hand
1 q1q 2 screw rule.
U=
4 0 k r12
7. For a surface under consideration, electric field
iii. For electric dipole in uniform field. 
  E is due to both inside as well as outside
U =  p E charges. But term (q) represents only the total
q charge inside closed surface.
18. C =
V 8. By introducing dielectric slab between plates of
19. For a parallel plate capacitor: charged capacitor, capacitance increases,
i. Without dielectric potential difference decreases and charge
0 A remains unchanged.
C=
d 9. If n plates are arranged as shown in figure
ii. With dielectric below, then they constitute (n1) capacitors in
0A parallel. The resultant capacitance of this
C= combination is given by,
 1
d  t 1   0 A
 k C = (n  1)
20. Capacitors in series: d
1 1 1 1 + + +
= + + + + + + +
CS C1 C2 C3 +  + +   + +   + + 
+  + +   + +   + + 
21. Capacitors in parallel: +   + +   + +   + + 
      
CP = C1 + C2 + C3
22. Energy stored in capacitor: 
1 1 1 q 2
10. If n capacitors each of same capacitance C, when
U= CV2 = qV =
2 2 2 C are connected in series then equivalent
23. Energy loss in capacitor: C
capacitance of combination is CS= and when
C1C 2  V1  V2 
2 n
U  U = are connected in parallel then equivalent
2  C1  C 2 
capacitance is CP = nC.
Shortcuts 11. When two capacitors are connected to each other
with wire, the charge on them is redistributed.
1. When two identical conductors having charges The ultimate potential of both capacitor is called
q1 and q2 are kept in contact and separated later common potential and is given by,
 q1  q 2  V1C1  V2C 2
then each has charge of   . If charges are V=
 2  C1  C 2
 q1  q 2  12. Energy spent by the battery in charging the
q1 and  q2, then, each has charge  .
 2  capacitor = qV,
2. Coulomb’s law is in accordance with the 1
whereas energy stored = qV.
Newton’s third law of motion. 2
 
i.e., Fq1 q 2   Fq 2 q1 13. The total energy stored in series or parallel
grouping of capacitors is sum of energies stored
3. It two charges q1 and q2 are separated by in individual capacitors.
distance r, then distance x between charge q, and Total energy U = U1 + U2 + U3 + .....
null point
r
14. Total energy stored per unit volume of the
x= capacitor,
q2
1 U 1
q1 u=  0E2
volume 2
4. The resultant electric field at a point due to
15. The energy stored in parallel plate capacitor is
various neighbouring charges is
1
   
E  E1  E 2  E 3  ........
U= k0E2Ad
2
19
Physics Vol‐II (Med. and Engg.)
9. When a body is connected to the earth,
Multiple Choice Questions electrons from the earth flow into the body.
This means the body is….. [K CET 2004]
1.1 Electric charges and their conservation (A) uncharged.
(B) charged positively.
1. Which one of the following is the unit of (C) charged negatively.
electric charge? (D) an insulator.
(A) coulomb (B) newton 10. Five balls numbered 1 to 5 are suspended
(C) volt (D) coulomb/volt using separate threads. Pairs (1, 2), (2, 4) and
2. The dimensional formula of electric charge is (4, 1) show electrostatic attraction while pairs
(A) [M0 L0 T1 A1] (B) [M0 L0 T–1 A1] (2, 3), (4, 5) show repulsion. Therefore, ball 1
0 0 1 –1
(C) [M L T A ] (D) [M0L0T–1A–1] must be [BCECE 2015]
(A) neutral
3. The electric charge always resides (B) metallic
(A) at the centre of charged conductor. (C) positively charged
(B) at the interior of charged conductor. (D) negatively charged
(C) on the outer surface of charged 11. Transfer of electric charge can take place in
conductor. such quantities which are integral multiples of
(D) randomly all over the charged (A) 1 e.s.u. of charge
conductor. (B) 1 coulomb
4. Which among the following is a sure test of (C) 1 micro coulomb
electrification? (D) 1.6  1019 coulomb
(A) Attraction (B) Induction 12. A glass rod rubbed with silk is used to charge
(C) Repulsion (D) Conduction a gold leaf electroscope and the leaves are
5. A body can be negatively charged by observed to diverge. The electroscope thus
[AIIMS 1998; C PMT 2009] charged is exposed to X-rays for a short
period. Then [AMU 1995]
(A) giving excess of electrons to it.
(A) the divergence of leaves will not be
(B) removing some electrons from it.
affected.
(C) giving some protons to it.
(B) the leaves will diverge further.
(D) removing some neutrons from it.
(C) the leaves will collapse.
6. One metallic sphere A is given positive charge (D) the leaves will melt.
whereas another identical metallic sphere B of 13. Which is bigger, one coulomb or charge on an
exactly same mass as of A is given equal electron?
amount of negative charge. Then (A) One coulomb
[AMU 1995; R PET 2000; C PMT 2000] (B) Charge on electron
(A) mass of A and mass of B still remain (C) Both are same
equal. (D) None of these
(B) mass of A increases.
14. When 1014 electrons are removed from a
(C) mass of B decreases. neutral metal sphere, the charge on the sphere
(D) mass of B increases. becomes [Manipal MEE 1995]
7. When a glass rod is rubbed with silk, it (A) 16 C (B) –16 C
[MP PET 2003] (C) 32 C (D) –32 C
(A) gains electrons from silk. 15. Number of electrons in one coulomb of charge
(B) gives electrons to silk. will be
(C) gains protons from silk. [MP PMT/PET 1998; Pb. PMT 1999;
(D) gives protons to silk. AIIMS 1999; R PET 2001]
8. Two identical conductors of copper and (A) 5.46  1029 (B) 6.25  1018
–19
aluminium are placed in an identical electric (C) 1.6  10 (D) 9  1011
fields. The magnitude of induced charge in 16. When 1019 electrons are removed from a
aluminium will be [AIIMS 2008] neutral metal plate through some process, the
(A) zero electric charge on it is
(B) greater than in copper [K CET 1999, GUJ CET 2015]
(C) equal to that in copper (A) –1.6 C (B) +1.6 C
(D) less than in copper (C) 10 C +19
(D) 1019 C
20
20 
Chapter 01 : Electrostatics
17. If a charge on the body is 1 nC, then how 25. In Coulomb’s law, the constant of
many electrons are present on the body? 1
proportionality C = has units
[K CET 2014] 4 0
(A) 1.6 × 1019 (B) 6.25 × 109 (A) N (B) Nm2
(C) 6.25 × 1027 (D) 6.25 × 1028 (C) Nm2/C2 (D) NC2/m2
18. A copper sphere of mass 2 g contains about
1
2  1022 atoms. The charge on the nucleus of 26. The magnitude of is
4 0
each atom is 29e. What fraction of the
electrons must be removed from the sphere to (A) 9  109 Nm2/C2
give it a charge of +2 C? [BCECE 2014] (B) 9  109 Nm2/C2
(A) 1.08  1011 (C) 8.85  1012 Nm2/C2
(B) 2.16  1011 (D) 8.85  1012 Nm2/C2
(C) 3.24  1011
27. The ratio of the forces between two small
(D) 4.32  1011
spheres with constant charge (a) in air (b) in a
19. In one gram of a solid, there are 5  1021 medium of dielectric constant k is
atoms. If one electron is removed from every [MNR 1998]
one of 0.01 % of atoms of the solid, charge (A) 1 : k (B) k : 1
gained by the solid would be (C) 1 : k2 (D) k2 : 1
(A) 0.08 C (B) 0.8 C
(C) –0.08 C (D) –0.8 C 28. When the distance between the charged
particles is halved, the force between them
20. A conductor has 14.4  10–19 coulomb becomes [MNR 1986]
positive charge. The conductor has (A) one-fourth
(Charge on electron = 1.6  10–19 coulomb) (B) half
(A) 9 electrons in excess. (C) double
(B) 27 electrons in short. (D) four times
(C) 27 electrons in excess.
(D) 9 electrons in short. 29. There are two charges +1 microcoulomb and
+ 5 microcoulomb. The ratio of the forces
21. Charge on -particle is [MH CET 2000] acting on them will be [C PMT 1979]
(A) 4.8  10–19 C (A) 1 : 5 (B) 1 : 1
(B) 1.6  10–19 C (C) 5 : 1 (D) 1 : 25
(C) 3.2  10–19 C
(D) 6.4  10–19 C 30. An electron is moving round the nucleus of a
hydrogen atom in a circular orbit of radius r.
1.2 Coulomb’s law  force between two 
The coulomb force F between the two is
point charges
1
(Where k = ) [CBSE PMT 2003]
22. The law governing the force between electric 4πε 0
charges is known as e2
(A) k rˆ
[C PMT 1972; MP PMT 2004] r3
(A) Ampere’s law e2 
(B) Ohm’s law (B) k r
r3
(C) Faraday’s law
(D) Coulomb’s law e2 
(C) k 3 r
r
23. For what order of distance is Coulomb’s law e 2

true? (D) k 2 rˆ
r
(A) for all distance.
(B) distance greater than 10–13 cm. 31. Two point charges placed at a certain distance
(C) distance less than 10–13 cm. r in air exert a force F on each other. Then the
(D) distance equal to 10–13 cm. distance r at which these charges will exert
24. Dimensions of 0 are the same force in a medium of dielectric
(A) [M0 L0 T0 A0] constant k is given by
(B) [M0 L–3 T3 A3] [EAMCET 1990; MP PMT 2001]
(C) [M–1 L–3 T3 A1] (A) r (B) r / k
(D) [M–1 L–3 T4 A2] (C) r / k (D) r k
21
Physics Vol‐II (Med. and Engg.)
32. Two equally charged, identical metal spheres 37. A total charge Q is broken in two parts Q1 and
A and B repel each other with a force F. The Q2 and they are placed at a distance R from
spheres are kept fixed with a distance r each other. The maximum force of repulsion
between them. A third identical, but
between them will occur, when
uncharged sphere C is brought in contact with
A and then placed at the mid-point of the line [MP PET 1990]
joining A and B. The magnitude of the net Q Q
(A) Q2 = , Q1 = Q 
electric force on C is R R
[UPSEAT 2004; DCE 2005] Q 2Q
(B) Q2 = , Q1 = Q 
(A) F (B) 3F/4 4 3
(C) F/2 (D) F/4 Q 3Q
(C) Q2 = , Q1 =
33. Two charges of equal magnitudes and at a 4 4
Q Q
distance r exert a force F on each other. If the (D) Q1 = , Q2 =
2 2
charges are halved and distance between them
is doubled, then the new force acting on each 38. When air is replaced by a dielectric medium of
charge is constant k, the maximum force of attraction
[DCE 2010] between two charges separated by a distance
(A) F / 8 (B) F / 4 (A) decreases k times.
(C) 4 F (D) F / 16 (B) remains unchanged.
34. Identify the wrong statement in the following. (C) increases k times.
(D) increases k2 times.
Coulomb's law correctly describes the electric
force that 39. A force F acts between sodium and chlorine
[K CET 2005] ions of salt (sodium chloride) when put 1 cm
(A) binds the electrons of an atom to its apart in air. The permittivity of air and
nucleus. dielectric constant of water are 0 and K
(B) binds the protons and neutrons in the respectively. When a piece of salt is put in
nucleus of an atom. water, electrical force acting between sodium
(C) binds atoms together to form molecules. and chlorine ions 1cm apart is
(D) binds atoms and molecules together to [MP PET 1995]
form solids. F FK
(A) (B)
K 0
35. Two spheres carrying charges +6 μC and
F F 0
+9 μC, separated by a distance d, experience a (C) (D)
K0 K
force of repulsion F. When a charge of
−3 μC is given to both the sphere and kept at 40. Dielectric constant of pure water is 81. Its
the same distance as before, the new force of permittivity will be [C PMT 1984]
repulsion is [K CET 2015] (A) 7.17  10–10 MKS units
(B) 8.86  10–12 MKS units
F
(A) (B) F (C) 1.02  1013 MKS units
3
(D) 7.52  10–10 MKS units
F
(C) (D) 3F
9 41. Two charges placed in air repel each other by
a force of 10–4 N. When oil is introduced
36. Two spherical conductors B and C having between the charges, the force becomes
equal radii and carrying equal charges in them 2.5  10–5 N. The dielectric constant of oil is
repel each other with a force F when kept [MP PET 2003]
apart at some distance. A third spherical (A) 2.5 (B) 0.25
conductor having same radius as that of B but (C) 2.0 (D) 4.0
uncharged is brought in contact with B, then 42. Two charges each of 1 coulomb are at a
brought in contact with C and finally removed distance 1 km apart, the force between them is
away from both. The new force of repulsion [D PMT 1999; C PMT 2010]
between B and C is (A) 9  103 newton
[AIEEE 2004] (B) 9  10–3 newton
(A) F/4 (B) 3F/4 (C) 1.1  10–4 newton
(C) F/8 (D) 3F/8 (D) 104 newton
22
22 
Chapter 01 : Electrostatics
43. Two electrons are separated by a distance of 50. Two fixed point charges + 4q and + q units are
1Å. What is the coulomb force between them? separated by a distance ‘x’. Where should a
[MH CET 2002] third point charge  q0 be placed for it to be in
(A) 2.3  10–8 N equilibrium?
(B) 4.6  10–8 N (A) Midway between the charges + 4q and
(C) 1.5  10–8 N + q.
(D) 2.8  10–8 N (B) At a distance 2x from the charge + 4q.
(C) At a distance 2x/3 from the charge + 4q.
44. Two charges each equal to 2 C are 0.5 m (D) At a distance x/3 from the charge + 4q.
apart. If both of them exist inside vacuum, 51. In figure two positive charges q2 and q3 fixed
then the force between them is
along the y-axis, exert a net electric force in
[C PMT 2001] the + x-direction on a charge q1 fixed along
(A) 1.89 N (B) 2.44 N the x-axis. If a positive charge Q is added at
(C) 0.144 N (D) 3.144 N (x , 0), the force on q1.
45. +2 C and +6 C two charges are repelling each i. y ii. y
other with a force of 12 N. If each charge is q2 q2
given –2 C of charge, then the value of the
force will be Q x
x
[C PMT 1979; Kerala PMT 2002] q1 q1 O (x, 0)
(A) 4 N (Attractive)
(B) 4 N (Repulsive) q3 q3
(C) 8 N (Repulsive) [NCERT Exemplar]
(D) Zero (A) shall increase along the positive x-axis.
46. The charges on two sphere are +7 C and (B) shall decrease along the positive x-axis.
–5 C respectively. They experience a force F. (C) shall point along the negative x-axis.
If each of them is given an additional charge (D) shall increase but the direction changes
of – 2 C, the new force of attraction will be because of the intersection of Q with q2
[R PET 2010] and q3.
(A) F (B) F / 2 52. Two positive point charges are 3 m apart and
(C) F / 3 (D) 2F their combined charge is 20 C. If the force
between them is 0.075 N, then the charges are
47. Two point charges +3 C and +8 C repel (A) 10 C, 10 C (B) 15 C, 5 C
each other with a force of 40 N. If a charge of
(C) 12 C, 8 C (D) 14 C, 6 C
–5 C is added to each of them, then the force
between them will become 53. Two charges, each equal to q, are kept at
[SCRA 1998; JIPMER 2000] x = – a and x = a on the x-axis. A particle of
(A) –10 N (B) +10 N q
mass m and charge q0 = is placed at the
(C) +20 N (D) –20 N 2
origin. If charge q0 is given a small
48. The force between two charges 0.06 m apart is displacement (y << a) along the y-axis, the net
5 N. If each charge is moved towards the other force acting on the particle is proportional to
by 0.01m, then the force between them will [JEE (Main) 2013]
become [SCRA 1994] (A) y (B) – y
(A) 7.20 N 1 1
(B) 11.25 N (C) (D) 
y y
(C) 22.50 N
(D) 45.00 N 1.3 Superposition principle, forces
between multiple charges
49. Two small conducting spheres of equal radius
have charges +10 C and –20 C respectively 54. A charge q1 exerts some force on a second
and placed at a distance R from each other charge q2. If third charge q3 is brought near,
experience force F1. If they are brought in the force of q1 exerted on q2 [NCERT 1971]
contact and separated to the same distance, (A) decreases.
they experience force F2. The ratio of F1 to F2 (B) increases.
is [MP PMT 2001] (C) remains unchanged.
(A) 1 : 8 (B) – 8 : 1 (D) increases if q3 is of the same sign as q1
(C) 1 : 2 (D) – 2 : 1 and decreases if q3 is of opposite sign.
23
Physics Vol‐II (Med. and Engg.)
55. Point charges + 4q , – q and + 4q are kept on 60. Three equal charges are placed on the three
the x–axis at point z = 0, z = a and z = 2a corners of a square. If the force between q1
respectively. and q2 is F12 and that between q1 and q3 is F13,
(A) Only – q is in stable equilibrium. F12
the ratio of magnitudes is [MP PET 1993]
(B) None of the charges are in equilibrium. F13
(C) All the charges are in unstable (A) 1/2 (B) 2
equilibrium. (C) 1/ 2 (D) 2
(D) All the charges are in stable
equilibrium. 61. Three charges each of magnitude q are placed
at the corners of an equilateral triangle, the
56. If charge q is placed at the centre of the line electrostatic force on the charge placed at the
joining two equal charges Q, the system of center is (each side of triangle is L)
these charges will be in equilibrium if q is [D PMT 2009]
(A) – 4Q (B) – Q/4 1 q2
(C) – Q/2 (D) + Q/2 (A) zero (B)
4πε 0 L2
57. Three charges each equal to 1 C are placed at 1 3q 2 1 q2
(C) (D)
the corners of an equilateral triangle. If force 4πε 0 L2 12πε 0 L2
between any two charges is F, then the net
force on either will be 62. Four charges are arranged at the corners of a
square ABCD, as shown in the adjoining
(A) 3F (B) 2F
figure. The force on the charge kept at the
(C) 3 F (D) F/3 centre O is [NCERT 1983; BHU 1999]
58. Which picture below best represents the A B
+q +2q
direction of the force on the charge at the
origin due to the other two changes shown in
figure? Y O
– 2q +q
+2Q D C
x (A) zero.
X (B) along the diagonal AC.
+Q x –Q (C) along the diagonal BD.
(D) perpendicular to side AB.
63. Three charges are arranged as shown in the
(A) (B) figure. The magnitude of the force on q2 due
to charge q1 is F21. The magnitude of the force
on q2 due to charge q3 is F23. The ratio F21/F23
is 3 C
q3

(C) (D)
2m

1 C 2 C
q1 q2
1m
59. Charges of + 10 C and  20 C are placed as
in figure. The force on a 5 C charge is (A) 2/3 (B) 4/3
directed to the right everywhere (C) 2 (D) 3/2
+10 C 20 C 64. Charge q1 = + 6.0 nC is on Y-axis at
I II III y = + 3 cm and charge q2 = – 6.0 nC is on
Y-axis at y = – 3 cm. Calculate force on a
(A) in region I. charge q0 = 2 nC placed on X-axis at x = 4 cm.
(B) in region II. (A) – 51.8 ĵ N (B) + 51.8 ĵ N
(C) in region III.
(D) in region II and III. (C) – 5.118 ĵ N (D) + 5.18 ĵ N

24
24 
Chapter 01 : Electrostatics
532. The difference in the effective capacitance of (A) 0 C (B) 5 C
two equal capacitors when joined in parallel (C) 10 C (D) 25 C
and series is 3 F. The value of each capacitor
is 537. Eight drops of mercury of equal radii and
possesssing equal charges combine to form a
(A) 1 F (B) 2 F
big drop. The capacitance of bigger drop as
(C) 3 F (D) 4 F compared to capacitance of each individual
533. Four capacitors each of capacitance 4 F are drop is
connected as shown in the figure. The energy [BCECE 2015]
stored in the system is (A) 16 times (B) 8 times
C (C) 2 times (D) 32 times
C C 538. The charge on a capacitor of capacitance
C 10 F connected as shown in the figure is
[AMU 1995]
2
10 V
3 10 F

(A) 5  105 J (B) 5J


(C) 8  105 J (D) 2J
534. Three identical capacitors are combined 2V
differently. For the same voltage to each (A) 20 C (B) 15 C
combination, the one that stores the greatest (C) 10 C (D) zero
energy is [MP PMT 1995; BCECE 2014]
(A) two in parallel and the third in series 539. In the circuit here, the steady state voltage
with it. across capacitor C is a fraction of the battery
(B) three in series. e.m.f. The fraction is decided by
(C) three in parallel. [AMU (Engg.) 2000]
(D) two in series and third in parallel with it. R1

535. Assertion: A parallel plate capacitor is C


charged by a battery. The battery is then R2
disconnected. If the distance between the R3
plates is increased, the energy stored in the
capacitor will decrease.
Reason: Work has to be done to increase the (A) R1 only.
separation between the plates of a charged (B) R1 and R2 only.
capacitor. (C) R1 and R3 only.
(A) Assertion is True, Reason is True; (D) R1, R2 and R3.
Reason is a correct explanation for
540. A combination of capacitors is set up as
Assertion.
shown in the figure. The magnitude of the
(B) Assertion is True, Reason is True; electric field, due to a point charge Q (having
Reason is not a correct explanation for a charge equal to the sum of the charges on the
Assertion.
4 F and 9 F capacitors), at a point distant
(C) Assertion is True, Reason is False.
30 m from it, would equal:
(D) Assertion is False, Reason is True.
[JEE (Main) 2016]
536. A capacitor of capacitance 5 F is connected 3 F
as shown in the figure. The internal resistance 4 F
of the cell is 0.5 . The amount of charge on
the capacitor plate is [MP PET 1997]
1 1  9 F

5 F 2 F + 
2
8V

2.5 V (A) 360 N/C (B) 420 N/C


+ – (C) 480 N/C (D) 240 N/C

65
Physics Vol‐II (Med. and Engg.) 
541. Which one statement is correct ? A parallel (A) 22  106 J
plate air condenser is connected with a battery. (B) 11  106 J
Its charge, potential, electric field and energy  32  6
are Q0, V0, E0 and U0 respectively. In order to (C)   10 J
 3
fill the complete space between the plates a
 16  6
dielectric slab is inserted, the battery is still (D)   10 J
3
connected. Now the corresponding values Q,
V, E and U are in relation with the initially 545. A 40 F capacitor in a defibrillator is charged
stated as to 3000 V. The energy stored in the capacitor
(A) Q > Q0 (B) V > V0 is sent through the patient during a pulse of
(C) V < V0 (D) U < U0 duration 2 ms. The power delivered to the
542. n identical condensers are joined in parallel patient is
and are charged to potential V. Now they are [AIIMS 2004]
separated and joined in series. Then the total (A) 45 kW
energy and potential difference of the (B) 90 kW
combination will be (C) 180 kW
[MP PET 1993] (D) 360 kW
(A) energy and potential difference remain 546. In air, a charged soap bubble radius of ‘r’ is in
same. equilibrium having outside and inside
(B) energy remains same and potential pressures being equal. The charge on the drop
difference is nV. is (0 = permittivity of free space, T = surface
(C) energy increases n times and potential tension of soap solution)
difference is nV. [MH CET 2014]
(D) energy increases V times and potential
2T0
difference remains same. (A) 4r2
r
543. 100 capacitors each having a capacity of 4T0
10 F are connected in parallel and are (B) 4r2
r
charged by a potential difference of 100 kV.
6T0
The energy stored in the capacitors and the (C) 4r2
r
cost of charging them, if electrical energy
costs 108 paise per kWh, will be 8T0
(D) 4r2
[MP PET 1996; D PMT 2001] r
(A) 107 joule and 300 paise.
(B) 5  106 joule and 300 paise. 547. If n drops, each of capacitance C and charged
(C) 5  106 joule and 150 paise. to a potential V, coalesce to form a big drop,
(D) 107 joule and 150 paise. the ratio of the energy stored in the big drop to
that in each small drop will be
544. Two capacitors C1 = 2 F and C2 = 6 F in [Assam CEE 2015]
series, are connected in parallel to a third
(A) n : 1 (B) n4/3 : 1
capacitor C3 = 4 F. This arrangement is then 5/3
connected to a battery of e.m.f. = 2 V, as (C) n : 1 (D) n2 : 1
shown in the figure. How much energy is lost 548. A particle of mass m and charge +q is midway
by the battery in charging the capacitors? between two fixed charged particles each
[MP PET 2001]
C1 C2 having a charge +q, and at a distance 2L apart.
The middle charge is displaced slightly along
the line joining the fixed charges and released.
The time period of oscillation is proportional
C3 to [Assam CEE 2015]
1/2
+ – (A) L (B) L
2V (C) L3/2 (D) L2

66
66 
Chapter 01 : Electrostatics

Answers to MCQ's
1. (A) 2. (A) 3. (C) 4. (C) 5. (A) 6. (D) 7. (B) 8. (C) 9. (B) 10. (A)
11. (D) 12. (B) 13. (A) 14. (A) 15. (B) 16. (B) 17. (B) 18. (B) 19. (A) 20. (D)
21. (C) 22. (D) 23. (B) 24. (D) 25. (C) 26. (A) 27. (B) 28. (D) 29. (B) 30. (C)
31. (C) 32. (A) 33. (D) 34. (B) 35. (A) 36. (D) 37. (D) 38. (A) 39. (A) 40. (A)
41. (D) 42. (A) 43. (A) 44. (C) 45. (D) 46. (A) 47. (A) 48. (B) 49. (B) 50. (C)
51. (A) 52. (B) 53. (A) 54. (C) 55. (C) 56. (B) 57. (A) 58. (C) 59. (C) 60. (B)
61. (A) 62. (C) 63. (B) 64. (A) 65. (C) 66. (B) 67. (C) 68. (C) 69. (D) 70. (A)
71. (B) 72. (C) 73. (C) 74. (A) 75. (B) 76. (A) 77. (D) 78. (D) 79. (C) 80. (C)
81. (A) 82. (A) 83. (B) 84. (A) 85. (C) 86. (B) 87. (D) 88. (D) 89. (C) 90. (C)
91. (A) 92. (D) 93. (C) 94. (C) 95. (D) 96. (D) 97. (B) 98. (C) 99. (A) 100. (B)
101. (C) 102. (C) 103. (B) 104. (C) 105. (C) 106. (B) 107. (B) 108. (B) 109. (C) 110. (A)
111. (C) 112. (C) 113. (B) 114. (A) 115. (A) 116. (A) 117. (A) 118. (A) 119. (A) 120. (A)
121. (A) 122. (B) 123. (D) 124. (D) 125. (C) 126. (C) 127. (D) 128. (B) 129. (C) 130. (B)
131. (C) 132. (B) 133. (C) 134. (D) 135. (A) 136. (A) 137. (B) 138. (D) 139. (C) 140. (A)
141. (A) 142. (B) 143. (A) 144. (A) 145. (A) 146. (C) 147. (D) 148. (C) 149. (C) 150. (C)
151. (D) 152. (C) 153. (A) 154. (D) 155. (A) 156. (B) 157. (B) 158. (B) 159. (B) 160. (C)
161. (A) 162. (A) 163. (C) 164. (A) 165. (D) 166. (A) 167. (C) 168. (A) 169. (C) 170. (C)
171. (D) 172. (A) 173. (C) 174. (B) 175. (B) 176. (D) 177. (B) 178. (C) 179. (C) 180. (D)
181. (A) 182. (B) 183. (D) 184. (B) 185. (C) 186. (C) 187. (B) 188. (B) 189. (B) 190. (D)
191. (B) 192. (C) 193. (D) 194. (A) 195. (A) 196. (C) 197. (B) 198. (C) 199. (B) 200. (B)
201. (A) 202. (A) 203. (A) 204. (D) 205. (A) 206. (B) 207. (C) 208. (C) 209. (C) 210. (D)
211. (C) 212. (C) 213. (B) 214. (A) 215. (C) 216. (C) 217. (A) 218. (B) 219. (C) 220. (C)
221. (A) 222. (B) 223. (D) 224. (D) 225. (A) 226. (C) 227. (B) 228. (C) 229. (D) 230. (B)
231. (C) 232. (C) 233. (C) 234. (B) 235. (D) 236. (B) 237. (A) 238. (D) 239. (C) 240. (D)
241. (A) 242. (D) 243. (B) 244. (D) 245. (D) 246. (B) 247. (C) 248. (C) 249. (B) 250. (D)
251. (A) 252. (D) 253. (A) 254. (C) 255. (C) 256. (B) 257. (D) 258. (C) 259. (D) 260. (C)
261. (C) 262. (C) 263. (C) 264. (D) 265. (A) 266. (D) 267. (C) 268. (C) 269. (D) 270. (B)
271. (B) 272. (D) 273. (C) 274. (A) 275. (D) 276. (C) 277. (C) 278. (C) 279. (D) 280. (B)
281. (B) 282. (D) 283. (B) 284. (C) 285. (B) 286. (D) 287. (B) 288. (A) 289. (C) 290. (C)
291. (C) 292. (D) 293. (C) 294. (C) 295. (A) 296. (A) 297. (A) 298. (B) 299. (A) 300. (B)
301. (B) 302. (D) 303. (B) 304. (A) 305. (D) 306. (D) 307. (B) 308. (D) 309. (D) 310. (B)
311. (D) 312. (C) 313. (C) 314. (D) 315. (A) 316. (A) 317. (B) 318. (D) 319. (D) 320. (D)
321. (A) 322. (B) 323. (A) 324. (A) 325. (B) 326. (A) 327. (C) 328. (B) 329. (B) 330. (A)
331. (C) 332. (A) 333. (C) 334. (A) 335. (B) 336. (A) 337. (B) 338. (C) 339. (B) 340. (A)
341. (D) 342. (B) 343. (A) 344. (B) 345. (B) 346. (A) 347. (D) 348. (B) 349. (B) 350. (B)
351. (A) 352. (D) 353. (B) 354. (C) 355. (B) 356. (C) 357. (D) 358. (D) 359. (D) 360. (D)
361. (B) 362. (A) 363. (C) 364. (D) 365. (D) 366. (C) 367. (A) 368. (B) 369. (C) 370. (C)
371. (D) 372. (D) 373. (B) 374. (C) 375. (A) 376. (C) 377. (B) 378. (D) 379. (D) 380. (C)
381. (C) 382. (C) 383. (D) 384. (B) 385. (A) 386. (D) 387. (B) 388. (C) 389. (B) 390. (B)
391. (C) 392. (A) 393. (A) 394. (A) 395. (C) 396. (D) 397. (C) 398. (B) 399. (B) 400. (D)
401. (A) 402. (C) 403. (C) 404. (B) 405. (B) 406. (B) 407. (D) 408. (C) 409. (B) 410. (C)
411. (C) 412. (A) 413. (D) 414. (B) 415. (D) 416. (D) 417. (B) 418. (B) 419. (D) 420. (D)
421. (B) 422. (A) 423. (C) 424. (C) 425. (D) 426. (C) 427. (B) 428. (B) 429. (C) 430. (D)
431. (A) 432. (B) 433. (B) 434. (C) 435. (D) 436. (C) 437. (B) 438. (C) 439. (B) 440. (B)
441. (C) 442. (C) 443. (C) 444. (A) 445. (A) 446. (A) 447. (D) 448. (B) 449. (A) 450. (B)
451. (A) 452. (B) 453. (B) 454. (D) 455. (D) 456. (B) 457. (A) 458. (D) 459. (D) 460. (A)
461. (D) 462. (C) 463. (A) 464. (C) 465. (C) 466. (B) 467. (C) 468. (C) 469. (C) 470. (C)
471. (B) 472. (B) 473. (A) 474. (C) 475. (D) 476. (A) 477. (A) 478. (B) 479. (B) 480. (A)
481. (C) 482. (C) 483. (A) 484. (B) 485. (C) 486. (A) 487. (B) 488. (B) 489. (A) 490. (B)
491. (B) 492. (B) 493. (C) 494. (A) 495. (A) 496. (A) 497. (D) 498. (D) 499. (D) 500. (C)
501. (C) 502. (C) 503. (A) 504. (A) 505. (B) 506. (D) 507. (C) 508. (D) 509. (B) 510. (C)
511. (A) 512. (B) 513. (B) 514. (A) 515. (C) 516. (A) 517. (D) 518. (C) 519. (B) 520. (D)
521. (A) 522. (C) 523. (A) 524. (A) 525. (B) 526. (A) 527. (D) 528. (B) 529. (C) 530. (C)
531. (A) 532. (B) 533. (C) 534. (C) 535. (D) 536. (C) 537. (C) 538. (A) 539. (B) 540. (B)
541. (A) 542. (B) 543. (C) 544. (B) 545. (B) 546. (D) 547. (C) 548. (C)

67
Physics Vol‐II (Med. and Engg.) 

Hints to MCQ's

2. Charge = current  time = AT = [M0L0T1A1] 18. Q = ne


Q
5. Excess of electron gives the negative charge  n=
on body. e
2 10 6
6. Negative charge means excess of electrons n=
2 10  29 1.6 10 19
22

which increases the mass of sphere B.


Whereas positive charge on sphere A is given  n = 2.16  1011
by removal of electrons. 0.01
19. n=  5  1021 = 5  1017
7. On rubbing glass rod with silk, excess 100
electrons are transferred from glass to silk. So q = – ne = – 5  1017  (–1.6  10–19) = 0.08 C.
glass rod becomes positive and silk becomes 20. Positive charge shows the deficiency of
negative. electrons.
8. Since both are metals so equal amount of 14.4×1019
Number of electrons = =9
charge will be induced in them. 1.6×1019
9. When a positively charged body connected to 21. By using, q = ne
the earth, electrons flows from earth to body q = + 2e = + 3.2  10–19 C
and body becomes neutral.
1 q1q 2
10. 24. F=
4πε 0 r 2
1 q1q 2
1 2 3 4 5 0 =
4πF r 2
= repulsion 1 [A 2T 2 ]
= 1 1 -2
[M LT ] [L2 ]
= attraction
= [M–1 L–3 T4 A2]
As 2 and 3 repel and also 4 and 5 repel; 2, 3, 4
1
and 5 must be charged. 25. Constant of proportionality C =
As 2 and 4 attract, they must be oppositely 4 πε 0
charged. Unit of 0 = C2 / Nm2
As 1 attracts both 2 and 4. This is possible 1 1 Nm 2
Unit of = 2 =
only for a neutral ball. Hence, 1 is neutral. 4  0 C / Nm 2
C2
12. Charge on glass rod is positive, so charge on { 4 has no dimensions}
gold leaves will also be positive. Due to
X-rays, more electrons from leaves will be q1q 2 q1q 2
27. Fa = ,Fb =  Fa : Fb = k : 1
emitted, so leaves becomes more positive and 4πε 0 r 2 k4πε 0 r 2
diverge further.
1
13. No. of electrons constituting 1 C, 28. F ; so when r is halved the force becomes
r2
q 1 four times.
n   0.625  1019
e 1.6  10 19
29. The same force will act on both bodies
14. q = ne = 1014 ×1.6 ×10 19 although their directions will be different.
q = 1.6  105 C = 16 C
 
  | F1 |   | F2 |
Electrons are removed, so charge will be
positive.  

 e2 e2   r̂ = r 
q 1
30. F = k rˆ =  k. 3 r
15. n= = = 6.25×1018 r 2
r  r
e 1.6×1019  

16. By using Q = ne 31. F = F


Q = 1019  1.6  1019 
q1q 2
=
q1q 2
Q = +1.6 C 4πε 0 r 2 4πε 0 r' 2 k
r
17. n=
q
=
1  10 9
= 6.25  109  r 
e 1.6  10 19 k
68
68 
Chapter 01 : Electrostatics
32. Initially, CQ1 (Q  Q1 )
Q Q From (i) and (ii), F =
r2
A B dF
r For F to be maximum =0
dQ1
q2 C d
F=C 2
r  0=
r 2 dQ1
 QQ1  Q12 
where C is constant of proportionality
 0 = Q  2Q1
Finally,
 Q = 2Q1
Q/2 Q/2 Q
F F B A Q
 Q1 =
A C B 2
r/2 r/2 Q
 Q2 =
2
When uncharged sphere C is brought in
1 q1q 2 F
contact with charged sphere A, the sphere C 38. F = =
4πε 0 k r 2 k
Q
acquires a charge due to conduction. If F is the force in air, then F is less than F
2
C(Q / 2) CQ2 2 since k > 1.
Force on C due to A, FA = = 2
(r / 2) 2
r 1 q1q 2
39. F=
CQ(Q / 2) 2CQ 2 40 K r 2
Force on C due to B, FB = =
(r / 2) 2 r2 1
 F
CQ 2 K
 Net force on C, Fnet = FB  FA = =F For air K = 1
r2
Let F be electrical force when placed in water
q2 F 1
33. F=C . If q is halved, r is doubled then  
r2 F K
q
2
F
C   F
2 Cq 2 1 Cq 2 F K
F  2  = 
 2r   4  4  r 2 16 r 2 16 40.  = k0 = 81  8.854  10–12
= 7.17  10–10 MKS units
34. Nuclear force binds the protons and neutrons
in the nucleus of an atom. Fa 104
41. k= k= =4
Fm 2.5×105
35. F  Q1Q2 ( r is same in both cases)
Cq 2 1
F QQ 6 9 42. F= = 9×109 ×12 × = 9×103 N
 = 1 2 = =3 r2 (1000) 2
F  
Q1Q 2 (6  3)  (9  3)
F q2
 F = 43. F = 9×109 ×
3 r2
(1.6×1019 ) 2
Q2 = 9×109 × = 2.3×108 N
36. Initially F = C (figure A). Finally when a (1010 ) 2
r2
third spherical conductor comes in contact q2
44. By using F = 9×109.
alternately with B and C then removed, so r2
charges on B and C are Q/2 and 3Q/4 (2 ×10 6 ) 2
respectively (figure B)  F = 9 ×109. = 0.144 N
(0.5) 2
Q Q Q/2 3Q/4
45. Resultant charges after adding the –2 C be
B
(–2 + 2) = 0 and (–2 + 6) = + 4 C
B C C
r r Cq1q 2 0× 4
 F= = C× 2 = 0
(A) (B) r2 r
 Q  3Q  1 (+7 ×106 )(5×106 )
   3 46. F=
Now force F = C  2 2 4  = F 4πε 0 r2
r 8
1 35×1012
= N
Q1Q 2 4πε 0 r2
37. Q1 + Q2 = Q ….(i) and F = C ….(ii)
r2

69
Physics Vol‐II (Med. and Engg.) 
1 (+5×106 )( 7 ×106 ) +y
F =
4πε 0 r2
+q2
1 35×1012
= N
4πε 0 r2
f = fnet + fqQ
 F = F x +x
-q1 +Q(x, 0)
47. In second case, charges will be –2 C and
+3 C
+q3
F q1q 2
Since F  q1q2 i.e., =
F q1q2 –y
40 3× 8
 = = 4 Now, if positive charge Q is added at (x, 0), it
F'  2 × 3 exerts an attractive force (fqQ) on q1, along
 F = –10 N + x-axis.
1 Thus, the force on q1 shall increase along the
48. F positive x-axis.
r2
F1  r2 
2
1 q1q 2
 =  52. F=
F2  r1  4  0 r 2
2 9  109 (q1q 2 )
5  0.04  0.075 =
 =  (3) 2
F2  0.06 
0.075  9
 F2 = 11.25N q1 q2 =
9  109
49. F  q1q2 = 0.075  10–9
When both spheres are brought in contact with = 75  10–12 ….(i)
each other, each sphere acquires a charge, Given, q1 + q2 = 20  10–6
10  20  q1 = 20  10–6 – q2
= 5 c
2 Putting in (i),
F1 q1q 2 10×  20 8 (20  10–6 – q2) q2 = 75  10–12
 = = =
F2 q1q2 5×  5 1  20  10–6 q2 – q22 = 75  10–12
 q22 – 20  10–6 q2 + 75  10–12 = 0
50. x
 q22 –1510–6 q2 – 510–6 q2 + 75  10–12 = 0
+4q (qo) (xa) +q  q2 (q2 – 1510–6)–510–6 (q2 – 1510–6) = 0
a  q2 = 15  10–6 or q2 = 5  10–6
 Two charges are 15 C, 5 C.
4q×q 0 q×q 0 53.
F= = Fnet
a2 (x  a) 2 F F
4(x  a)2 = a2 
2(x – a) = a
2x – 2a = a 
2x y
3a = 2x  a =
3 q q/2 q
O
51. +y a a

+q2 Fnet = 2 F cos 


2  kq 2
x
fnet
+x =  cos 
q1 2  a 2  y2 

+q3 kq2 y
= 
 a  y2 
2
a 2
 y2 
y
As, net force on charge q1, due to positive For y << a
charges q2 and q3 is along + x-axis. Thus, it is kq 2 y
F
an attractive force. a3
Hence, q1 must be a negative charge. Fy
70
70 
Chapter 01 : Electrostatics
q 1q 2 64. Here, q =  6.0 nC =  6.0  10–9 C
54. The force will still remain according
4  0 r 2 2 a = 6 cm = 6  10–2 m
to the superposition principle. r = 4 cm (on equatorial line)
= 4  10–2 m
56. For equilibrium, net force on Q = 0 q0 = 2 nC = 2  10–9 C,
CQQ CqQ q1
  2 0
(2x) 2 x
 q = – Q/4  5
3
57. Each charge experiences two forces inclined at
60 . Therefore, 4
0 q0
R= F2 + F2 + 2FFcos60o = 3F 2 = 3F F2 F1
3 5 F1 cos + F2 cos
59. F  q1q2 FL  (5) (+10) = 50 
FR  (5) (20) = +100
q2
As FR > FL In I  Left or Right
depends on distance. F = F1 cos + F2 cos
In II  Always Left 1 qq 0
=2 cos
In III  Always Right 40 r 2
60. q2 a q1 6  10 9  2  10 9 3
= 2  9  109  
(5  10 2 ) 2 5

2a = 5.184  10–5 N
a 
 F = 51.8ˆj N

q3 65. Net force on B, Fnet  FA2  FC2


2 2
1 q 1 q A +15 esu
F12 =  2 and F13 = 
4πε 0 a 4πε 0 (a 2) 2
F12 3 cm
 =2
F13
B FC C
   +12 esu 4 cm –20 esu
61. In the following figure since | FA |  | FB |  | FC |
and they are equally inclined with each other, Fnet  FA2  FC2
FA
so their resultant will be zero.
15  12 12  20
q FA   20 dyne , FC   15 dyne
 3  4
2 2
A
 Fnet  FA2  FC2  (20) 2  (15) 2  25 dyne
Q 66. FA = force on C due to charge placed at A
FB FC
FA 106  2  106
 9  109   1.8 N
q q (10  102 ) 2
C B
FB = force on C due to charge placed at B
62. We put a unit positive charge at O. Resultant 106  2  106
force due to the charge placed at A and C is  9  109   1.8N
(0.1) 2
zero and resultant force due to B and D is FA
towards D along the diagonal BD. For a unit +2C
negative charge, resultant force is towards B C 120o
along BD.
FB
qq
63. F  12 2
r
+1C – 1C
1× 2 3 2 3
 F21  and F23  2
= A 10 cm B
1 (2) 2
F21 2 4 Net force on C,
 = =
F23 (3 / 2) 3 Fnet  (FA )2  (FB )2  2FAFB cos120o  1.8N

71
Chapter 01 : Electrostatics
503. 2
O A Electric field at point B, E = E 2x  E 2y =
40r
T cos 
 Ey
T
 tan  = 1
q Ex
qE =
T sin  2 0   = tan1(1) = 45

 E makes an angle of 45 with AB.
mg
A R 2 102  5 106
505. E = =
 0 kr 0.40 1
T cos  = mg ….(i) 10  108
q
= = 9  109  10–6
T sin  = ….(ii) 40  101  1
20
= 9  103 N/C
Dividing equation (ii) by equation (i),
q 506.
tan  =
20 mg

504. Taking into account the horizontal component


of electric field at point B, a
dEx = dE cos  Q
r dr
dx x
A

Electric field due to charge Q at r = a is,
r
r x
2 2 kQ
Ea = ….(i)
B a2
dEx Consider a shell of thickness dr in the region
dE a  r  b.
dEy
A
charge on shell, dq = Area   = 4  r2d r
( dx) x r
 dEx =  total charge in the region a  r  b is,
40 (r 2  x 2 ) r 2  x2
b b b
   r2 
x q =  dq = 4A  r dr = 4A  
 From figure, cos     2 a
 r x 
2 2 a
2 2
a

 xdx  q = 2A [b  a ]
dEx = Electric field at r = b is,
4 0 (r 2  x 2 )3/ 2
 k 2  A  b2  a 2   Q
 xdx Eb = ….(ii)
 Ex =  dE x   b2
40 0 (r 2  x 2 )3/ 2

For electric field to be constant in the region
 1 a  r  b we must have,
 2x(r  x ) dx
2 2 3/ 2
Ex =
40 2 0 Ea = Eb
Let r2 + x2 = t From equations (i) and (ii),
 differentiating w.r.t. t, 2x dx = dt kQ 2 A  b2  a 2   Q
   k  
a2 b2
  (r  x 2 )3/ 2 2 x dx = t
2 3/2
dt
Q b2
 Q  2 A  b2  a 2 
0 r2

On integrating we get, 2/r a2
 1 2 Q b 2  Qa 2
 Ex =   2 A  b2  a 2 
4  0 2 r a2
 Q  b2  a 2 
 Ex =
4  0 r  2 A  b2  a 2 
a2
 Q
Similarly, EY =  dE Y   dE sin    A=
4  0 r 2 a 2

101
Physics Vol‐II (Med. and Engg.) 
507. When the particle lies in the plane of the ring 4 2 Q2 2Q 2
where no net force acts on it, then it must lie at W   Q(V  VO )  QVO  
40a 0a
the centre of the ring.
Suppose, the particle is displaced slightly from 511 Potential at the centre of square,
mean position by x (say).  9  109  50  106 
Electric field due to the ring at the new V  4    90 2  10 V
4

 2/ 2 
position of the particle is given by
(2a) x Work done in bringing a charge (q = 50 C)
E= from  to centre (O) of the square is
40 (a 2  x 2 )3/ 2
W  q(V0  V )  qV0
x
For x < < a, E = ….(i)  W  50 106  90 2 104  64J
20a 2
(directed away from centre of ring) Q
Now, force acting towards the centre of the 512. Electrical potential, V =
4 0 R
ring is given by
F = – qE Electric field E = 0 inside a conductor
 x  514. At centre E = 0 and V = 0
 F = – q 2
….[using (i)]
 20a  A B
+Q –Q
 x  E E
ma = – q  2
(F = ma)
 20a  E O E
 q 
 a=–  2
x ….(ii) –Q +Q
 20ma  D C
Also, equation for SHM is given by  1 q
a = – 2 x ….(iii) 515. Potential at the centre is 5   
Comparing equations (ii) and (iii),  40 l 
q The electric field due to the oppositely placed
2 = charges cancel and net electric field is
20ma 2
 1 q
q  2 
.
 = ….(iv)  40 l 
20ma 2
Hence, time period, 516. We know that potential at surface of a sphere
2 20ma 2 q
T= = 2 ….[using (iv)] is V = K = 10 V (Given) ….(i)
 q r
where, q and r is the charge and radius of the
508.  K.E. = W = Fx = qEx small drop respectively.
509. From charge configuration, at the centre, As the volume of 27 small drops and that of 1
electric field is non-zero. Potential at the large drop must be same, hence
2q 4  4
centre due to 2q charge V2q  27  r 3  = R 3 
r 3  3
2q where, R is radius of large drop.
 R = 3r
r
Now the total charge on large drop is
r r Q = 27q
E– q E2q E– q Hence, potential at surface of this drop is
–q –q
KQ (27q) Kq
and potential due to – q charge V = = K =9
R (3r) r
q
V q   (r = distance of centre point)  V = 90 V ….[Using eq (i)]
r
 Total potential V  V2q  Vq  Vq  0 517. V = 6x – 8xy – 8y + 6yz
V
510. Potential at centre O of the square Ex =  = (6 – 8y) = 2
x
 Q  V
VO  4   Ey =  =  (8x  8 + 6z) = 10
 4 0 (a / 2)  y
Work done in shifting (– Q) charge from centre V
Ez =  = 6y = 6
to infinity z
102
102 
Chapter 01 : Electrostatics

E= E 2x  E 2y  E 2z = 4  100  36 = 140 3 30
ii. Quartz:   7.5 ;
0.4 4
= 2 35 N/C 2
F = qE = 2  2 35 = 4 35 N iii. Teflon:  2 ;
1
 5 50
518. E= 
dV iv. Mica:  6
dr 0.8 8
  
r = xi + y j + zk


526. E =   = k0E
k 0

V  V  V 
E =  i j z   = 2.2  8.85  1012  3  104
x y z
 6  107 C/m2
 At point (1, 1, 0);
       527. If nothing is said, it is considered that battery
E =  6 i  5 j  2 k = (6 i +5 j + 2 k )
is disconnected. Hence charge remains the
1 2
same.
E A 2 mVA WA (q)V   t
519.    Also Vair   d and Vmedium   d  t  
EB 1 0 
mVB2 WB (4q)V 0 k
2
 t
dt 

VA 1
 Vm  k
VB 2  
Va d
520. From conservation of energy,  6
86 
 P.E =  K.E. Vm  6
 
 Ui  Uf = (K.E.)f  (K.E.)i 120 8
q1 q 2  1 1  1  Vm = 45 V
    = mv f
2
….{ (K.E)i = 0}
40  ri rf  2  0 Ak
528. C  4  0 r
2 q1 q 2  1 1  d
 v f2     r = Radius of sphere of equivalent capacity
m 40  ri rf 
Ak 100  104  6
2 1106  2  103  9 109  1 1   r   4.77 m
=    4 d 1  103  4  3.14
1103  1 10 
529. Electric field inside parallel plate capacitor
v f  32400
2

having charge Q at place where dielectric is


 vf = 180 m/s Q
absent = and where dielectric is present
  A 0

AB =  B  A  = a 4jˆ  3kˆ
 
521.
  Q
=
  ˆ kA 0
 
 
 Work done = F AB = q   k  a 4jˆ  3kˆ
 20  C1 1
3qa     530. =
 ….{ k  j  0 ; k  k 1 } C2 3
2 0 In series, charge on each capacitor is same.
V1 3
522. In the given condition angle between p and E
 
 = q / C1 = C 2 =
V2 q / C2 C1 1
is zero. Hence potential energy Dividing 10 V in the ratio 3 : 1, we get
U   pE cos 0   pE  min. V1 = 7.5 V, V2 = 2.5 V
Also in uniform electric field Enet = 0.
531. On connecting O at A, 4 F capacitor is
1 1 charged to a constant potential (E).
523. C  k; V  ; U 
k k As connection of O is switched over to B, the
524. High k means good insulating property and total charge on 4 F capacitor is shared
high x means able to withstand electric field 4 2
gradient to a higher value. between 4 F and 2 F capacitor is =
42 3
k of original charge.
525. C ;
d 532. CP = C1 + C2
4 40 1 C  C2
i. Glass:   20 ; = 1 + 1 = 1
0.2 2 CS C1 C2 C1C 2

103
Physics Vol‐II (Med. and Engg.) 
C1C 2  R = 2r
 CS =
C1  C 2 the capacitance of bigger drop is
According to given condition C = 40R = 40 (2r) = 2C
CP – CS = 3 F 538. In steady state potential difference across
C1C 2 capacitor = 2 V.
C1 + C2 – =3
C1  C 2 So charge on capacitor Q = 10  2 = 20 C
As C1 = C2 = C 539. In steady state potential difference across
C2 capacitor
2C – =3
2C V2 = potential difference across resistance
4C2 – C2 = 6 C  R2 
R2   V
3 C2 = 6 C  R1  R 2 
C = 2 F Hence V2 depends upon R2 and R1.
1 1 1 1 R2
533. For total capacitance = + + R1
C C CC C
1 1 1 2 1 2 5 C R3
= + + = = V1
4 8 4 8 8
V2
C = 8/5 F and V = 10 V
Then energy
1 1 8
= CV2 =   10–6  (10)2 +
V

2 2 5
540. Cnet = 5 F
= 8  10–5 J.
 Qnet = 5  8 = 40 C
1 We know,
534. U CV 2
2 Q2F = 2  8 = 16C
Now if V is constant, then U is greatest when  Q4F = Q12F = Qnet  Q2F
Ceq is maximum. This is when all the three are
...{9F|| 3F = 12F}
in parallel.
= 40  16 = 24C
535. The charge q remains unchanged as the battery
is disconnected. The capacitance C decreases Voltage across 4F and 12F can be given as,
if the separation between the plates is V4F + V12F = V
increased. Now, energy stored U = q2/2C. Q12F 24
 V4F = V  =8 = 6V
Since q remains the same and C is decreased, C12F 12
U will increase.  V12F = 2V
536. In steady state condition, no current flows i.e. V9F = 2V
through line (1). Hence total current  Q9 F = 9  2 = 18 C
2.5  Q = Q4 F + Q9 F = 42 C
I   1A
(1  1  0.5) kQ 9  109  42  106
 E=  = 420 N/C
1 1 r2 30  30
Line (2)
541. Capacitance will be increased when a
5 F 2 dielectric is introduced in the capacitor but
Line (1) potential difference will remain the same
because battery is still connected. So
according to
q = CV, charge will increase i.e., Q > Q0 and
2.5V
1 1
Potential difference across line (2) = potential U= QV0 , U 0  Q 0 V0
2 2
difference across capacitor
 Q > Q0 so U > U0
= 1  2 = 2 volt
So, charge on capacitor = 5  2 = 10 C 542. According to energy conservation, energy
remains the same
537. Volume of 8 drops will be same as volume of
1 1 C
1 large drop formed by combining smaller  Uparallel  Useries  (nC)V2    V2
drops. 2 2 n 
4  4  V = nV
 8  r 3   R 3 (V = potential difference across series
3  3
combination)
104
104 
Chapter 01 : Electrostatics
1 1  Knq 
543. Energy stored in the capacitor  CV 2  100  Q  1/ 3 
2 U 2 QV QV n r
   
1 6 U 1 qV  Kq 
  10  10 ×(100  103 ) 2  100  5  106 J qV q 
2 2  r 
Electric energy costs = 108 paise per kWH U nq Knqr
 
108 paise U n1/3 r q K q

3.6  10 6 J U n 5/3
5  106  108  =
 Total cost of charging  = 150 paise U 1
3.6  106
548. 2L
CC 2 6  
544. Ceq  1 2  C3   4  5.5 F F2 m F1
C1  C2 26 q1 = +q q2 = +q
Energy supplied (E)  qV  CV2  22 106 J +q
x
1 1 Displaced position
P.E.stored (U)  Ceq V 2 =  5.5  106  (2) 2 of particle
2 2 

U = 11  10 6 J Let F2 be the force exerted by charge q2 on +q



 Energy lost  E  U  11106 J Let F1 be the force exerted by charge q1 on +q
1  Fnet = F2  F1
CV 2
1  40  10 6  (3000) 2 kq 2 kq 2
545. Power = 2 = = 90 kW = 
t 2  2  10 3 (L  x) 2
(L  x)2
546. For the soap bubble,  1 1 
Pin  Pout = Pexcess = PST  Pelectro = kq 2   
 (L  x) 2
(L  x)2 
4T q2 4T q2
=  2 =   (L  x)2  (L  x)2 
r 2A 0 r 2(4r 2 )2 0 = kq 2  2 
 (L  x) (L  x) 
2

4T q2
=   L2  x 2  2Lx  L2  x 2  2Lx 
r 322r 40 = kq 2  
 (L2  x 2 )2 
For equilibrium,
Pin = Pout  4Lx 
2 = kq2  2 2
q  x ) 
2
4T (L
 =
r 322 r 40  4Lx 
= kq 2  4 
….{ x << L}
16  2  4 r 0T
2 4
16  8 r 0T
2 4
 L 
 q= =
r r  4x 
= kq 2  3 
80T L 
 q = 4r2
r 4kq 2 x
Fnet =
L3
547. Let the charge of each drop be q
q  Fnet = ma
 C=
V 4kq 2 x
 = ma ….(i)
 q = CV L3
 charge of final drop, Q = nq  a = 2 x (for S.H.M) ….(ii)
Radius of each small drop is r and big drop is R 2
4kq x
 volume remains same,  2 x =
mL3 ….from (i) and (ii)
4 3 4 3 4kq 2
 R = n  r  2
 =
3 3  mL3
3 3
R = nr 4kq 2
R = (n)1/3 r =
mL3
Potential on big drop 2
KQ  T=
V = 
R
mL3
Knq T = 2
V = 1/3 4kq 2
n r
Ratio of energy stored in big drop to small drop, T L3  T  (L)3/2
105
Physics Vol‐II (Med. and Engg.) 

Topic Test
1. A charge of 5.8  1018 C 8. A charge Q is enclosed by a Gaussian
(A) does exists. spherical surface of radius R. If the radius is
(B) does not exist. doubled, then the outward electric flux will
(C) exists for a short time. (A) increase four times.
(D) exists only if placed in higher orbits. (B) be reduced to half.
(C) remain the same.
2. Two point charges repel each other with a
(D) be doubled.
force of 100 N. One of the charges is
increased by 10% and other is reduced by 9. Two charged conducting spheres of radii R1
10%. The new force of repulsion at the same and R2 separated by a large distance are
distance would be connected by a long wire. The ratio of the
(A) 100 N (B) 121 N charges on them is
(C) 99 N (D) 90 N R1 R2
(A) (B)
3. Three charges – q1, + q2 and – q3 are placed as R2 R1
shown in figure. The x component of the force R12 R 22
(C) (D)
on – q1 is proportional to R 22 R12
–q3 y
10. The electric potential due to the nucleus of the
hydrogen atom at a distance of 5.3  10–11 m is
27.2 V. What is the potential due to the helium
a nucleus at the same distance?
 (A) 27. 2 V (B) 54.4 V
b (C) 13.6 V (D) 20.4 V
–q1 x
+q2 11. The electric field in a region is radially
outward with magnitude E = Ar. Find the
q 2 q3 q 2 q3 charge contained in a sphere of radius 20 cm.
(A)  sin  (B)  cos 
b2 a 2 b2 a 2 Given: A = 100 V m–2
q 2 q3 q 2 q3 (A) 9.89  10–11 C
(C)  sin  (D)  cos 
b2 a 2 b2 a 2 (B) 7.85  10–11 C
4. For corona discharge, the conductor should (C) 8.89  10–11 C
have a (D) 8.19  10–11 C
(A) spherical end (B) flat end
12. Two positive point charges 12 C and 8 C
(C) pointed end (D) concave end
are placed 10 cm apart in air. How much work
5. A square is centered at origin having sides must be done to bring them close by 6 cm?
parallel to x and y-axis. It has surface charge (A) 2.8 J (B) 3.8 J
density  (x, y) = 0 xy within its boundaries. (C) 4.8 J (D) 13.0 J
The measure of charge of each side of square
13. The radius of a spherical conductor is 0.9 m.
is Q. What is the total charge on the square?
Its capacity is
(A) 40Q2 (B) 20a3
2 (A) 1 pF (B) 10 pF
(C) 0a (D) Zero
(C) 100 pF (D) 1000 pF
6. Two positive charges of 20 coulomb and Q
14. The capacitance of the earth viewed as a
coulomb are situated at a distance of 60 cm.
spherical conductor of radius 6408 km is
The neutral point between them is at a
(A) 600 F (B) 712 F
distance of 20 cm from the 20 coulomb
charge. Charge Q is (C) 980 F (D) 1424 F
(A) 30 C (B) 40 C 15. When two capacitors are connected in series,
(C) 60 C (D) 80 C 15
the equivalent capacitance is F. When
7. An electric dipole, when held at 30 with 4
respect to a uniform electric field of 104 NC–1 they are connected in parallel the equivalent
experiences a torque of 9  1026 Nm. capacitance is 16 F. The individual capacitance
Calculate dipole moment of the dipole. are
(A) 1.7  10–29 Cm (B) 1.8  10–30 Cm (A) 5 F, 11 F (B) 6 F, 10 F
(C) 1.8  10 Cm–29
(D) 1.7  10–30 Cm (C) 4 F, 12 F (D) 8 F, 8 F
106
106 
Chapter 01 : Electrostatics
16. The capacities of two conductors are C1 and
C2 and their respective potentials are V1 and
V2. If they are connected by a thin wire then
the loss of energy will be
C1C 2 (V1  V2 )
(A)
2(C1  C 2 )
C1C 2 (V1  V2 )
(B)
2(C1  C 2 )
C1C2 (V1  V2 )2
(C)
2(C1  C2 )
(C1  C 2 ) (V1  V2 )
(D)
C1C 2

17. If a point charge q0 is placed at the centre of a


thin wire ring of radius r, then what is the
increased value of force exerted on stretching
the wire? [Given, q is the electric charge on
the wire.]
qq 0 qq 0
(A) (B)
42 0 r 2 22 0 r 2
qq 0 qq 0
(C) (D)
2 0 r 2 8 2  0 r 2

Answers to Topic Test


1. (B) 2. (C) 3. (C) 4. (C)
5. (D) 6. (D) 7. (C) 8. (C)
9. (A) 10. (B) 11. (C) 12. (D)
13. (C) 14. (B) 15. (B) 16. (C)
17. (D)

107

Das könnte Ihnen auch gefallen